Licensure Exam

Réussis tes devoirs et examens dès maintenant avec Quizwiz!

A young family just migrated to US with little financial support. The social worker wants to let family know about family resources that may help them with this transition. The social worker should first: A) find out if family understands language B) find out if the family is ok with accepting resources C) tell the family resources are limited but available D) call several agencies and make the referrals for them

Answer A. Be sure they understand and communicate first

A teenage boy was be screened for admission to a mental health facility reporting that he was experiencing low self-esteem, failure to perform school work, and pessimism. He also reports difficulty sleeping that has been lasting about a year. This is most likely: A) dysthymia B) conduct disorder C) bipolar disorder D) oppositional

Answer is A, dsythymia. **

Viewing the world "only" thru one culture one culture is? A) ethnocentric B) prejudice C) understandable D) ego centric

Answer is A, ethnocentric. Egocentric means self importance. Ethnocentric means your culture is superior to others.

A client has been recently admitted to an inpatient care unit. Soon after his arrival, he starts demanding to be released. The client asks the social worker to help get him out of the hospital as he wants to be discharged right away. According to the treatment team he is not ready to be discharged anytime soon. The worker should: A) listen to the client's requests and explain his rights for discharge B) tell the client he can't be discharged as he is not ready C) immediately contact the client's family D) take no action and forward the information to the unit administrator

Answer A. Every client that is admitted, voluntary or involuntary has rights for discharge. With a voluntary client they can sign out Against Medical Advice, without involuntary admissions they have the right to request to see the judge. Generally, court is actually held at the hospital where involuntary admissions are reviewed periodically. This is a good possibility. Now look at the other three answers. B. Key Words: "TELL the client". We would not tell the client he cannot be discharged. If he is voluntary, he can sign out AMA, this is his right. If he is involuntary he can request to see a judge for a hearing and competency etc. will be determined. C. We would not contact the family of an adult, and if we did, how would this help the client? He can still discharge himself AMA. D. Key words "take no action." We have to take action. Something must be done to help the client.

A social worker is assigned to a 12 yr. old boy who is a disruptive in school. In the first session w/ the boy, the, the boy states his family has been homeless for several months. The social worker should. A) contact the local welfare agency B) use behavior modification practices with this boy C) contact the parents to recommend disciplinary skills D) utilize listening skills and offer support

Answer A. IF the child is homeless he and his parents are living in an unsafe environment and mandatory reporting would constitute A. A Better answer is to further clarify the child's situation before action is taken, but that is not a choice here.

A group of social workers are meeting. The supervisor of the group needs to pick one social worker from the group to represent them on an issue. The supervisor asks for input from the group and one of the individuals recommends picking a male as they know more. What should the supervisor do? A) confront the supervisee and ask her why she thinks males know more than females B) bring up the issue of gender bias and open it up for discussion in the group C) ignore the comment and pick someone based on seniority D) let the group vote on who will go

Answer B. Address gender bias in the group and see what others know about it. The group can be a wonderful way to educate other group members.

When planning on reuniting foster child with his/her family, what should the social worker focus on? A) remind family the reasons that child was removed from home B) focus on status of risk factors C) assist the family to relocate D) help the parents get marriage counseling

Answer B. Always focus on the risk factors and keeping the child safe.

A social worker works in an emergency room. She works very long 12-hour shifts. She has a co-worker who is constantly late to relieve her from her shift at work. What should the social worker do first? A) report the co-worker to the supervisor B) confront the social worker C) announce it in staff meeting D) leave at your scheduled time

Answer B. Generally, we confront before we refer. The best choice is to first confront the social worker about his or her behavior. In an ethical situation this is the first response. This behavior could become an ethical issue if a client is harmed because of lateness. FIRST confront the colleague and if that does not get a response and change in behavior report to the supervisor. Allow the social worker to take corrective action first. The two best choices are A and B. Get it down to two choices and make your best educated guess.

A social worker is working with a male client that is in the middle of a divorce. He reports that his wife is claiming he has an alcohol problem that impairs his performance as a parent. The social worker has been seeing this client for several weeks and has seen no evidence of any impairment or problems with his judgment ability. He asks the social worker to state this in court as they decide custody issues. What should the social worker do next? A) tell the client to go to alcoholics anonymous B) submit statement to court C) refer him for psychiatric evaluation D) tell him to fight the custody and represent himself

Answer B. The social worker can submit a statement to the court of what she has seen. This is not a problem. If a written statement is not enough, the social worker can serve as an expert witness in court. Of the choices B, is the NEXT response.

A four year old boy whose father often travels away from home tells his son goodbye. When the father leaves the boy cries and says to his mother that daddy is not coming back and begins to cry. The child does not understand that his father will be back after his trip even after the father reassured him so. What normal developmental stage is this child experiencing? A) sensimotor stage B) preoperational thought C) concrete operational D) formal operations

Answer B. This child is in the preoperational thought stage where children are between the ages of 2-7 and when it is mastered they realize irreversibility. Once a child has mastered this stage the child will know that dad will come home when his trip is over. He may miss him and not want him to go, but he knows Dad will come home.

A mother brings in her teenage Native American daughter to a session with a social worker. The young girl as recently tried to commit suicide and is depressed. The mother tells the social worker she wants to allow her daughter to attend a healing service with a medicine man. What should the social worker's response be? A) allow the mother to take the child and immediately offer services after ceremony B) tell the mother the child needs to be assessed for admission before the ceremony C) ask the mother why she will not allow immediate services for her daughter D) tell the mother that you will seek to press charges if she does not get her daughter immediate attention

Answer B: We always try to respect the culture. In this case, however, the daughter has tried to harm herself and she may still be a danger to self. Before the cultural experience the assessment should be offered immediately and to increase compliance honoring the culture is important. But since the daughter has tried to harm herself recently select answer B as it is closest to getting her immediate help.

When evaluating a client's progress, the social worker's most effective strategy would be to: A) seek client's feedback about the intervention B) select measurements which are easy to use C) gather primary post intervention data D) use pre and post measuring tools

Answer is A, get the information from the client.

When a clinical supervisor reviews the supervisees case documentation in the record related to a client, she is surprised to see the social work supervisee add a personal comment in the note about the client. What should the social work supervisor do? A) tell the supervisee she is doing well B) tell the supervisee not to take notes without the supervisor reviewing what she writes C) discuss the agency rules and review the policy on note-taking D) tell her this is unethical and must be changed at once

Answer C is best. Although D may be possible depending on the comment it is best to make sure she knows agency and professional record keeping procedures before you address other issues.

A school social worker has just completed a session with a teen age girl, who has requested assistance from the worker to arrange for an abortion. The worker should first: A) call the girls parents B) call to find out what there states laws involving teen abortions and parenting laws are C) assist and respect the teens' decision and supply the information requested D) refer the teen to a teen abortion center for counseling

Answer C. Generally family planning allows this. So I would say refer her for the service she needs.

A client support advocate is upset that the court granted visitation to mother after the mother let the child be abused. The parents' rights are terminated but they allow visitation with the mother. The advocate feels this is not in the best interest of the child, what should the advocate do: A) ask court to stop visitation B) recommend that child contact mother when she makes 18 C) support the wishes of the court D) monitor the mother's behavior reporting only negative responses to the child

Answer C. If the court grants it you can do nothing and obey the court.

A social worker is seeing a depressed man who brings his wife in to a session. The wife reveals she is a mental health professional and has done her own evaluation of the husband and wants to read what the social worker has written. The husband says nothing. What should the social worker do? A) don't allow it B) give the information she requests to the wife C) clarify the role of the wife in her husband's treatment D) interpret the wife's question for the couple

Answer C. In this case clarify the role of the wife as she is not a therapist as in her own situation she cannot be objective.

A client reports to a social worker in a mental health clinic that he has recently tested positive for HIV. He has not told his wife of the illness, and it has been three weeks since he was given the test results. When asked about his behavior he states he has not taken precautions with his wife because he just cannot believe this is real. It appears to the social worker that his denial is so great that he simply does not want to deal with the implications of the positive test and although aware of the need for safe sex practices, he is ignoring the information in hopes it will just "go away." In this counseling session, the best thing for the social worker to do is: A) inform the client that action will be taken to inform the proper authorities of his communicable disease B) call his wife and tell her C) encourage him strongly to tell his wife D) tell him you plan to invite his wife in for a joint session to assist him in breaking the news

Answer C. In this case the best practice principle is to get him to tell her by encouraging him to do so. It also appears he needs counseling and support in this area and although not mentioned in this question a referral for counseling/case management in this area is indicated.

A social worker is working with a client that is the mother of a 17-year-old boy. The mother is concerned and is seeking help on how to best motivate her son. She states her son does not want to get out of bed and sleeps often. He also refuses to take his medications and has been diagnosed by a physician with major depressive disorder. He also has an older brother who is diagnosed with schizophrenia and also lives with his mother. The client expresses that she is overwhelmed and concerned that she will "never be free" and will have to take care of both her sons for the rest of her life if she does not take action for the son that suffers from depression now. What should the social worker do? A) invite the son suffering from depression to come in for joint therapy B) have the mother sign a release giving you permission to call the doctor of her son with depression C) help mother develop a reward program for her depressed son D) explore the mothers concerns and focus attention on the other son as he is complicating the situation

Answer C. The best answer from these choices is C, as it most closely reflects the goals of the mother and addresses her immediate concern. It is possible the son could be included in the session but first help the client in the session. Second choice answers are A, D, although it says "explore" the answer is not exploring the problem as presented.

A client is concluding treatment at a family counseling agency. The client feels very appreciative of the social worker's service. At the end of the interview, the client offers a substantial monetary gift to the social worker in addition to paying the fee to the agency. The social worker should: A) accept the gift, acknowledging the client's contribution to treatment. B) refuse the gift, basing the action on ethical standards of practice C) accept the money but with the understanding that it will be donated to a local charity. D) refuse the personal gift and suggest that the client make a donation to the agency instead.

Answer in this question is B. Answer is between B and D. We simply cannot take monetary gifts from clients for ethical reasons. You say that the study materials you saw said the answer is A. I say no...Not in this case and the code of ethics is pretty clear on this one. Not sure what study guide you were using but I would disagree if they gave the answer as A. Really the best thing to do is refuse the gift on ethical reasons. If I knew for sure it was a non-profit agency I would say B is the better answer but the question does not say that.

A social worker is working with a 19 year-old male client that has poor eye contact. He keeps looking away from the social worker and his eyes are bloodshot and red. He is aloof and does not seem concerned about the session and requests to leave. He has sun glasses and puts them on and says the light is bothering him. What substance is this male client most likely abusing? A) marijuana B) cocaine C) aerosol sprays D) methamphetamine

Answer is A for marijuana. Answers B and D are both stimulants so they really do not fit. The aerosol sprays is a possibility but again the adolescent's behavior is generally very erratic so again it does not fit.

To enhance the client's capacity of decision-making the worker should: A) involve the client in working on solutions B) setting priorities for the client C) examining alternatives D) help the client to make the decision

Answer is A, always involve the client in the decision making as this is much more powerful for achieving long-lasting changes.

Based on the criteria for Schizophrenia, which type of Schizophrenia is most likely to exhibit a lack of systematized delusions and silly affect: A) disorganized type B) catatonic type C) paranoid type D) residual

Answer is A, this is most likely the disorganized type, not the paranoid, where delusions are very systematized. Catatonic is more of the waxy flexibility and movement issues.

What mental health condition is most likely to be treated with benzodiazepines? A) anxietyB) obsessive compulsive disorder C) psychosisD) depression

Answer is A.

A client offers babysitting services to the social worker in exchange of counseling services, because she is no longer able to pay. What should the social worker's response be? A) refuse the offer and discuss the possibility of providing a sliding fee scale to the client B) accept the offer C) arrange a different type of service that the client can provide D) refuse the offer and tell the client this is unethical

Answer is A. A sliding fee scale should be offered. It is not considered ethical so D is also an offer but A is more complete as it offers and acceptable plan.

A male transgendered client is caught using the female restroom. The female patrons are very upset. What should the social worker do first when this situation is brought to her attention and she is asked to handle it? A) Tell the client that he needs to use the male restroom. B) Put in a unisex restroom. C) Ask him how much longer will his treatments take D) Tell the client that he needs to decide which restroom he prefers and the patrons will have to adjust

Answer is A. A unisex restroom would be ideal, however, this is not practical. If there was one already available you could ask the client to use that one.

The social worker is hired to work in the office of the public defender. In this position the social worker is assigned to work with a teenager who was charged with auto theft. The worker does a background check to see if he has a close relationship with his family members. The teen responds "What's it to you?". Then just stares ahead in space. The social worker should: A) acknowledge the anger ad recognize the client is in a serious position B) explain why the information was needed C) explain the consequences of not responding to the worker D) ignore the teens comment and move on to the next question

Answer is A. Acknowledge feelings before you ignore or explain.

A social worker is working with a client that has been mandated by the court to seek treatment related to the alleged sexual abuse of his step-son. The client tells the social worker he does not want to come to the sessions, as he is innocent. What should the social worker do first? A) acknowledge his frustration with having to attend B) ask him if he wants another social worker C) ignore the frustration and address his resistance D) tell the client firmly he has no choice as he is court ordered

Answer is A. Acknowledge his frustration with attendance. This will help to establish rapport.

A couple is being seen in therapy by a social worker for martial counseling. The couple reports that they fight all of the time. While they are describing the fights, while talking about the incidents both are smiling. What should the social worker do? A) let the couple know that you are aware they smile when discussing the conflict B) ask the clients to enact one of the conflicts to help them become aware of the behavior to make them aware of their behavior. C) do nothing and forget about the smiling D) ask the couple to stop smiling

Answer is A. Acknowledge that you are aware of this behavior and that it seems incongruent. The question does not mention culture but this could be culturally elated. B, does not really fit as we acknowledge/explore before we intervene with action. C and D do not fit. Best choices are A and B, with B, being the better of the two choices. This could be a question that falls under the communication section of the test.

In an initial session, an adult client reports her life is a mess and mentions several family issue that are causing her problems such as her children, financial, marital and problematic parents. The technique that most likely supports hearing what the client is saying and responding accordingly is: A) active listening B) partializing C) empathizing D) advising

Answer is A. Active Listening: Active listening is a structured way of listening and responding to others. It focuses attention on the speaker. To begin this process, active listening allows the worker to suspend her own frame of reference and suspend judgment allowing full attention to the speaker. Partializing: Looking at problems that demand immediate attention as to those that can be put off for a time. Empathizing: responding to the emotional state or ideas of another person advising: offering advice, counsel and suggesting action. **

A social worker is running a therapeutic group and one of the clients complains that he is not having enough time to talk about issues that are important to him as others are "hogging" all the time. In addition, he feels that other members are not being sincere and addressing the real issues reflective of the purpose of the group. What should the social worker group facilitator do? A) ask what the other group members think B) tell the client to meet with the facilitator individually to address his concerns C) help the group to better process the agenda D) ask the group if others think the individual is correct

Answer is A. Always give the power to the group. This is always our philosophy unless danger to self or others.

A social worker is subpoenaed to give a deposition in court. What is the first thing the worker should do to prepare for this? A) get the client to sign a written release of information B) prepare to tell the court only what is requested C) tell the agency administrator what you plan to release D) ask the attorney what you can say

Answer is A. Always try to get a release from the client, even if you are subpoenaed to court. The other answers may be important considerations but get client permission first.

Which is most responsive to cognitive-behavioral therapy? A) Anxiety related thoughts and behaviors B) Substance abuse C) Borderline personality disorderD) Conduct Disorder

Answer is A. Answer A is best, followed by C. Borderline personality disorder, however, seems to respond best to dialectical behavioral therapy.

A client is in the process of a divorce. He states that his relationship with his wife is very strained. They have regular arguments about the children, the bills and the car payments and these problems seem to be escalating. The client reports he is afraid he will loose control and may not handle his anger toward her. What should the social worker do first? A) assess what the client means by losing control B) warn the wife of the danger C) call the police D) tell the agency administrator.

Answer is A. First assess before you do any of the other responses.

A social worker has a choice to do group therapy work or individual case work. A) the worker chooses group work because it offers a richer learning experience for the client. B) the worker chooses case work because the social worker could always do group work later. C) the worker chooses case work because group work is far more stressful and much more paper. D) the worker chooses case work because this method offers a greater degree of professional training for the field.

Answer is A. Answer a, looks like a good choice, as group work can offer a richer experience for the client if you want socialization or social influences that only a group can provide. This choice has the client's best interest in mind. B. This is not good for the client. You choose a method because it is best for the client, not more convenient for the social worker. The choice is always what is best for the client. C. Again, this is not a reason to pick one method over the other, where is the client's best interest? This is related to the convenience of the social worker. D. Again, this answer is for the social worker and the profession and does not have the interest of the client at heart. We work for the client, and start where the client is, not the profession

A social worker is working with a couple in therapy. The wife states that she had an affair because her husband is a truck driver and he was gone all the time. The social worker suggests the wife go on the road with her husband from time to time. The husband agrees with this and so does the wife. In this scenario the social worker is: A) manipulating the therapeutic session B) the social worker is placing her values on them C) discouraging independent decision making D) empowering the husband to have more control

Answer is A. Based on the choices provided, the best response is A, as the social worker is manipulating the therapeutic session.

A 13-year old Native American female is seen by a social worker. In the assessment phase, when discussing her culture, the female client becomes distant and appears disinterested. How should the social worker respond? A) acknowledge that discussing her culture may make her feel uncomfortable B) refer her to a group with adolescents that are of her own culture C) ask her if she wants a referral to another social worker D) tell her you are open for discussion when she is ready

Answer is A. Be sure to first acknowledge that culture may be a problem or barrier to communication before you take any other action.

A social worker is has recently started working with a family that has immigrated to the US with little resources. What is the first thing the social worker should do to help the family? A) determine whether the family can understand what the social worker is saying B) find out if the family's culture will allow them to accept things from others C) seek outside help for the family D) refer the family immediately for medical assistance

Answer is A. Before you can do anything else you have to determine if the family can understand what you are saying. Once done, you can work with the family on other needs.

At the last session a client made sexual comments towards the social worker. What should the social worker do prior to the next session? A) seek consultation with his/her clinical supervisor B) prepare by talking with another colleague what to say C) prepare to tell the client this is unethical D) refer the client to another social worker that is older and less likely to be a target of such attention

Answer is A. Best answers are A and B, but B is clearly better. Always seek consultation with your clinical supervisor.

A social worker is working with a client with a history of manipulation often using suicidal ideation and intent. When providing a safety plan for this client the social worker develops a detailed plan that requires examining what she is thinking and feeling and saying to herself as well as having the client call an emergency room or 24-hour hotline before they self-injure. What kind of therapy is being used with this client: A. Dialectical behavioral therapy B. Cognitive behavioral therapy C. Interactionist therapy D. Rational emotive therapy

Answer is A. Dialectical behavioral therapy is a type of interactive cognitive behavioral therapy often used with Borderline Personality Disorder and setting boundaries and limits in suicide. All of these choices are close but it is actually dialectical behavioral therapy a very specific type of behavioral therapy that is often used in prevention.

A social worker goes to a local church. She sees that one of her clients also is a member of the church. What should the social worker do first? A) discuss boundaries with the client B) the social worker should find another church C) ask the client to find another church D) refer the client to another social worker

Answer is A. Discuss boundaries first. Once done you may need to refer her to another social worker to prevent the potential for a conflict of interest

A graduate student is in the class of a social work educator. The educator also has a private practice. The graduate student asks the social work educator for a referral in the community for some personal issues and wonders if the educator can see them in practice. The best response for the educator is: A) give the student a list with at least three different names on it B) refer the student to his/her own practice C) tell the student it is unethical to do so and the student should find their own referral source D) ask the student what the problem is so you can better understand the situation

Answer is A. Due to a potential conflict of interest it is better to simply refer the student. The second option would be D, but it is better not to ask questions about the situation as it is of a personal nature and may have nothing to with the academic relationship. In addition, knowing this information may be problematic as the educator generally has the power of a grade.

A teacher refers an African American student to a social worker because he is loud and threatening in class. The student is only one of two African American students in the class. The student tells the social worker the teacher is racist. What should the social worker do? A) acknowledge the student is angry and explain policy regarding racism B) tell the student this is not racism C) tell student this is about his behavior, talk about situation D) address issues of racism with the teacher

Answer is A. Educate before you facilitate or intervene. Acknowledge anger and than educate the student to what racism is.

Individuals in a family are all seeing different social workers from different agencies. The best way to ensure the best care for your client is: A) explain the patient's rights to confidentiality B) make sure that all of the therapy providers maintain open communication C) ensure the family is in family therapy D) let the client state what they want for treatment

Answer is A. Explain confidentiality and empower the client.

A reports that she is having a problem with rat infestation and water leaks in her apartment complex and she does not know what to do. What should the social worker do first to help the client? A) tell the client to call the landlord first B) encourage the client to form committees in the complex to address the conditions C) recommend the client call the board of health D) ask the client if she has paid her rent

Answer is A. First the client should contact the landlord. if this does not work a call to the health department would be the next step.

A social worker is working with a client. The client's partner has recently become bedridden and needs total care due to an illness. The client is angry and says she did not sign up to be a caretaker for her husband. What should social worker do first? A) validate the client's feelings B) offer home health care assistance C) ask the client if she has any supports for herself D) provide referrals for placement for the partner

Answer is A. First validate the client's feelings. Once done, you may want to address the other issues.

A social worker is working with a 40-year-old client with no mental health history who had experienced several recent losses, decreased appetite, weight loss, insomnia and reports having auditory hallucinations telling him to kill himself. What is this client most likely suffering from: A) major depressive disorder with mood congruent psychosis B) anxiety disorder with agoraphobia C) catatonic schizophrenia D) dissociative disorder

Answer is A. From the choices it cannot be answer C although he is hearing voices, this sometimes occurs in bereavement and severe depression as they are delusional in nature and related to killing himself and suicide. The symptoms sound like depression with the auditory hallucinations attached. Also, the age is not typical of onset for schizophrenia.

A client is planning to leave for college for the first time in two weeks. He is reports having a hard time with family and his siblings and is fighting with them all the time. He asks the social worker what is wrong with him. What is the best response for the social worker? A) explain to the client that some people have reservations about leaving their family and this may be related to his recent problems B) acknowledge that he is depressed about leaving his family C) help the client establish a plan to work on his attitude problems D) tell the client this is normal and this will pass on its own

Answer is A. From the choices the best one is to help the client understand this may be a normal reaction that some people have and this may be related to his own recent behaviors.

Your client reports to you that she wants to kill her ex-boy friend who cheated on her. She has a plan and seems serious about carrying out her plans. A) public Law "duty to Warn" requires that you to report this case B) according to the Social workers code of ethics you are required to report the case C) this is a judgment call D) not reporting the case places the worker in full responsibility of the client's action, once she has been told.

Answer is A. Generally all social workers and other helping professionals honor duty to warn.

A client comes into the office, and tells the social worker that her boyfriend has hit her. She explains that after he hit her he apologized and promised never to hit her again. What is probably going to be the next cycle of violence? A) he will hit her again B) he will try to kill her C) he will tell her family she deserved it D) he will never hit her again

Answer is A. He will hit her again and often the violence escalates. Kill her is a possibility but generally not at this stage. The other two choices, C and D are unlikely and do not fit in the cycle of violence. generally they do hit again and the violence escalates.

A 43 year-old male client reports to the social worker in the first session he is feeling sad, lost and alone. His wife died two years ago and he finds little pleasure in activities and he cannot go hunting which he used to love doing. He also reports he has no friends and feels like he is dying. He went to a physician who gave him a prescription of sleeping pills and blood pressure medications. What should the social worker do? A) assess for suicide B) refer for antidepressant medications C) explore for history of substance abuse D) assess for complicated bereavement

Answer is A. IN this case the two best choices are A and D. Since he has pills and is clearly depressed assess for suicide ideation and intent first. Once done you can explore whether he has complicated bereavement as much of what he is feeling appears to be related to the death of his wife. The other two choices B and C do not fit.

A client is at the final session with her social worker and is preparing to terminate. The client invites the social worker to her wedding which will take place in a couple of weeks. What should the social worker's response be? A)tell her it would be inappropriate to attend based on ethical guidelines B)attend the wedding to show your support of her progress made in counseling C) accept the invitation, but do not show up D)ask her why she wants the social worker to attend

Answer is A. In terms of ethical guidelines it is understood "once a client always a client" and it would be better not to attend.

In an initial counseling session with a social worker, a client says that he is concerned regarding his anger issues. He reports that recently after a minor confrontation he felt like killing his neighbor. He states, however, that he does not have a plan and then retracts his threat. What should the social worker do first? A) assist the client in acknowledgment of his feelings of anger B) immediately contact police C) explain to the client duty to warn and that you will report this D) do nothing and continue the session

Answer is A. In this answer you start the process with acknowledging his feelings of anger Key words in A) assist/acknowledge anger B) contact police (not threatening at the time, need to assess further before this one) C) explain duty to warn and reporting (if he is danger to person with anger issues be careful as he could hurt social worker) D) do nothing- we never do nothing.

A new social work graduate is just starting her practice. She has never done individual therapy before and is getting ready to see her first client. As a supervisor, what would be most effective for her? A) role play individual therapy B) give literature relating directly to individual therapy C) assign readings on the subject D) give her information to give to the client

Answer is A. In this case role play with the supervisor is best. In addition, it would be good for the supervisor to role model for her and let her sit in on the session but this is not a choice listed above.

Which of the following is the best treatment group for individuals that have been survivors of sexual abuse? A. any type of treatment with others that have been abused B. family treatment, including the abuser C. motivational interviewing and direct confrontation D. psychotherapy to uncover repressed memories

Answer is A. In this question, we are not sure of the age of the individual or the gender, generally as adults groups can be very helpful, as a child, we need to be careful, doing the B, C and D as theses choices can be problematic. A is the best choice.

A school social worker is working with a student that reports considering acting on the suicidal ideations she has been having lately. Her parents are currently going through a divorce. What should the social worker do first? A) initiate hospitalization B) call parents for a meeting to discuss a family meeting C) notify the principal D) gather information from her teachers about her behavior

Answer is A. Initiate hospitalization. If it said she was considering suicide the family meeting would be a good choice.

The severity of an initial Major Depressive Episode is a predictor of: A) the potential for future reoccurrence B) how well the medication will work C) how well therapy will work D) will trigger a manic episode

Answer is A. It can be indicative of the potential for reoccurrence

L. is brought to the intake unit with police escort. He is psychotic and aggressive and has a history of drug abuse. After talking with the social worker on intake he agrees to be hospitalized voluntarily but after 24 hours he decides he wants to leave. He again becomes aggressive and states he wants to leave involuntarily. What should the social worker do? A) encourage him to stay voluntarily B) immediately seek involuntary placement for him C) refer him immediately to detoxification unit D) explain his rights and support his decision to leave

Answer is A. It is always best to get a client to stay voluntarily. If he does not want to stay and you have tried to get a voluntary admission, if you feel he is a danger to self or others, seek involuntary admission.

A client has missed several appointments with the social worker. She has also been late for several as well. The social worker over-hears the client telling the secretary that she is afraid to come in because she felt the social worker would be angry with her. What should the social worker do at the start of the session? A) discuss directly with the client her missing sessions and being late B) ask the secretary to come in and repeat what the client stated C) refer the client to another social worker D) ask the client not to discuss professional matters with the agency secretary

Answer is A. It is best to discuss this immediately with the client so the concerns can be addressed.

A social worker is working with a 10 year-old obese child that is very sad and does not have any friends. The child's parents have not been bringing him regularly to the sessions and he is often late or misses sessions. The social worker has met with the child six times and each time he has been late. The child seems to be enjoying the sessions and says he wants to keep coming. What should the social worker do? A) request a family session B) talk with the child's teacher C) ask to see the parents at home D) refer to a physician

Answer is A. It is best to have a family session to see what is causing the attendance problems.

An example of a functioning community is: A) congruent B) catchiness C) mental health provided D) complacent

Answer is A. It is congruent with what needs to be done and how this will be accomplished.

A foster family is interested in accepting a young child into their home for temporary placement. The young child is available for placement and suffers from several medical conditions including AIDS. What should the social worker reveal to the foster family where the child will be residing? A) inform the foster parents of the medical conditions including AIDS and what this will mean for care in the home B) do not release this protected health information to the foster parents C) place the child in the home telling the foster parents to see a medical professional to discuss health related information D) do not tell the parents but leave the child's record that has his positive status where the foster parents can see it

Answer is A. It is important to inform prospective foster parents about potential infectious diseases or other related health information as part of the placement. All foster parents need to be informed that the child has the disease and how to best handle the child's needs.

A social worker is working with a male client that is Mexican. She has been seeing the client for three months. He tells the social worker that he is feeling depressed and thinks that a witch has cast a spell on him. What should the social worker do first? A) assess him for depressive paranoia B) consider a folk healer to come in to help C) consider referring him to a medical doctor to help D) terminate him immediately.

Answer is A. It is quite possible this is culturally related but you must pick from the answers they give you. The best choice is to assess that it is not something else (mental health related) before you take the step to get a folk healer. Always ASSESS before you REFER if it is relevant.

Which is the most appropriate medication used in the treatment of bipolar disorder? A) lithium B) paxil C) zoloft D) alprazolam

Answer is A. Lithium

A social worker is assigned a Korean family who is concerned about their mother who is unable to care for herself. The family is thinking about placement. When the social worker goes to the house, the family does not communicate any problems and displays minimal eye contact. This is probably due to: A) the family practicing a cultural norm by not discussing family problems in front of strangers B) the family not wanting services C) the family feeling the social worker is intruding on their privacy D) the family is hiding the potential for abuse to the relative

Answer is A. Many times Asian families are very much closed to outside assistance until they feel comfortable with the person.

What is the most important consideration for a non-profit agency budget? A) fund-raising B) staff salaries C) client billing D) private profit-making

Answer is A. Non-profit agencies cannot make private profits and must put profits back into the agency. From these choices fund-raising seems most relevant.

The police bring a young male into the crisis clinic. He is hyperventilating and upset-after witnessing a shooting. Police want to question the male but are unsure if he is able to answer questions. What should the social worker do next? A) refer the client see a physician B) acknowledge person's feelings C) assess for PTSD D) assess whether the client is alert and oriented

Answer is D. Assess whether the client is alert and oriented.

A social worker decides she needs to take action to protect a client that is seeing an "active" alcohol abusing colleague. She reports her colleague to the ethics board. What is the reason for reporting this? A. to protect clients that the social worker will be working with B. an ethical requirement to report the co-worker C. a legal requirement to report the co-worker D. to protect the agency from a malpractice situation

Answer is A. Reporting of colleagues is ALWAYS done to protect the client(s) that could be harmed by the work of an impaired provider.

A supervisee comes to work in dressed in a non-professional way that violates the agency dress code. This is a new employee but she was given the policy manual when she was hired and her manner of dress is in direct conflict with the agency dress code. What should the supervisor do? A) give the employee a policy manual and discuss the dress code B) verbally confront the client about her dress in the next session C) tell the employee to go home and change her clothes D) ask the employee why she is violating the agency dress code

Answer is A. Review the policy with the employee

Which medication has a more paradoxical affect? A) Ritalin B) Prozac C) Navaine D) Benadryl

Answer is A. Ritalin is a stimulant but actually with the child that has ADHD it relaxes and helps the child to concentrate.

A woman is in therapy with the social worker reports several medical issues including back and shoulder pain. She states she is in so much pain all the time she has to take medications repeatedly and is considering taking her own life to avoid the pain. She reports she has attempted suicide in the past by overdose. The first course of action for the social worker is: A) initiate hospitalization B) refer to medical doctor for treatment C) complete a no-harm no-risk agreement D) help the individual to directly discuss and address her problems

Answer is A. She has a history of suicide, and a plan, so you need to seek hospitalization and worry about other things later.

A client reports to a social worker in the first session she is having a hard time with making friends. She states she cannot keep relationships and usually ends her relationships with a broken heart. The client comes to the second session with family photos that she wants to show the social worker. What should the social worker do? A) look at the photos as she requests B) tell the client that she has to set goals and objectives for the session C) ask the client to show her the photos at the end of the session D) tell her that the social worker is not her friend and there is work to be done

Answer is A. Start by looking at the photos quickly and respecting the clients need to share this with you to establish rapport.

A group of social workers working together in an agency are worried because a different person than their regular supervisor will be conducting their performance evaluation. The person conducting the evaluation is very good friends with another group of social workers in the agency. The social workers being evaluated are concerned because they think the person doing the evaluation will give more favorable ratings to her friends. What should the concerned social workers do? A) assume that the evaluator will follow the procedure and be fair B) ask for someone else to do the evaluation C) report their feelings of favoritism D) refuse to allow the evaluation to take place

Answer is A. The best course is to assume the evaluator will be fair. In addition, although it is not a choice, you would keep paper copies of the evaluation and if it did not exist in the future advocate for more objective criteria to be used if it was not already there. The objective criteria will help to keep the evaluation task focused and avoid the potential for the "halo" effect.

An example of role reversal would be? A) seven-year-old girl repeatedly comforting and reassuring her distressed mother following a marital separation B) nine-year-old girl sharing her mother's concerns about household bills C) single mother relying on her ten-year-old son to take on the responsibilities of the man of the house D) eleven-year-old boy demanding from his mother that his meals be on the table at a certain time

Answer is A. The child is comforting the mother.

A client is being seen in therapy for depression. Her cousin died six months ago. The cousin knows that her cousin was being seen by the social worker and at times the cousin drove her to the appointments at the agency. The current client asks about the deceased cousin's similarities in their diagnoses. The social worker first response should be: A) explain that ethical guidelines uphold confidentiality even after death B) do not confirm or deny the deceased as a client C) discuss the deceased with the cousin, as you believe it could help the client's current situation D) tell the client that her cousin was not your client.

Answer is A. The first response is to let the client know what ethical guidelines are but also we never deny or confirm whether a client is being seen or has been seen. Since the client already knows the client was seen by the social worker A makes the best course of action.

A homeless man has been referred to a social worker from a local hospital emergency room. The client is in his 40's, and shows obvious signs of psychosis and mental illness. He states he has not received any medication in over two weeks. What school of practice should be used by the worker? A) problem-solving B) psychosocial C) case work D) ecological

Answer is A. The key here is the client needs to get back on his medications, especially if he is psychotic. Find the model that will best address that. Start with what it is not. Answer C. Disregard this as case work versus group work really does not fit with the other choices which are schools or methods of practice. Answer is D. Two best choices are between A and B. From these two choices problem-solving is my first choice as it will help you address the need for medication very quickly. Psychosocial is also a possibility as it looks at the person-in-situation but it just seems like something more focused is needed. If crisis intervention were a choice I would pick this first.

A social worker sees a client on an initial interview complaining of the following symptoms for the past 2 years. The client reports a decreased desire in activities, loss of appetite and hypomanic episodes. Which diagnosis below best fits these symptoms? A) cyclothymia B) bipolar II C) dysthymia D) major depression

Answer is A. The key to this question is the two year duration and the depressed mood as well as the hypo-manic mood.

A 22 year-old female has a love/hate relationship with her Mother. She feels that no matter how hard she tries she has never had her Mother's approval. Her mother recently died and now the client tells the social worker in a session that she feels guilty about her Mother's death. What best explains the relationship this client had with her mother? A) enmeshment B) symbiotic C) detachment D) borderline personality disorder

Answer is A. The two best choices are enmeshment and symbiotic. Symbiosis: a relationship of mutual benefit or dependence. Enmeshment: this term comes from family systems theory and refers to a condition where two or more people weave their lives and identities around one another so tightly that it is difficult for any one of them to function independently. Detachment: this is the other opposite of enmeshment where the people are so independent in their functioning that it is difficult to figure out how they are related to one another. For the most part a healthy relationship is thought to be described by the space between enmeshment and detachment.

A social worker is working with a client on changing a problematic behavior. The worker implements a single subject design to record and monitor the effects of the treatment she is using. What is the social worker most likely trying to complete? A) a process evaluation B) a long term outcome study C) a cross-sectional approach D) a cohort study

Answer is A. The worker is most likely completing a process evaluation of what is happening with the client. It could also be considered an outcome evaluation but that is not a choice given.

A single Mom is raising three children on her own. She also has a mother that suffers from Alzheimer's disease and is in need of her care and supervision. This circumstance is often referred to as the: A) sandwich generation B) parent-child interaction problem C) divorce adjustment D) peter Pan Complex

Answer is A. This is an example of what is often referred to as the sandwich generation where the adult caregiver is expected to take care of their own children but at the same time their own parent needs care as well.

When developing a "means tested" program what is the most important factor to consider? A) income B) gender C) age D) contributions

Answer is A. This is exactly what means-tested means and the person must be eligible and generally this starts with income.

If a client brings up a major topic at the end of a session, this is most likely an example of: A) doorknob communication B) attention seeking behavior C) client trying to seek control D) manipulation of the social worker

Answer is A. This is the classic example of a door-knob communication. Generally if it is danger to self or others it needs to be addressed immediately. If not, the issue can be addressed at the next session.

A social worker is working with an elderly client that continually asks her to repeat everything she says. The client begins to get frustrated that the social worker is not speaking clearly. What should the social worker suspect? A) the client may have a hearing impairment B) the client may need extra support with understanding the directions. C) the client may be in the early stages of dementia D) the client may have socialization difficulties

Answer is A. This is typical of someone that is trouble hearing.

The mother of a three-year-old child is concerned that the child is seen interacting with an imaginary friend. What response should the social worker imitate? A) tell the mother this can be a very normal part of human growth and development B) refer the child to a physician for a complete physical before responding C) explore whether the child has early signs of schizophrenia D) assess for whether the child has been sexually abused

Answer is A. This may be considered a normal part of human growth and development.

A group of female social workers are constructing a group therapy program specifically designed for victims of rape. The best school of theory to design this group program after would be. A) psychosocial B) psychoanalytical C) psychosexual D) cognitive

Answer is A. Throw out B and C as they are the same thing and would not work. This leaves A and D. Most of these programs are psychosocial in nature. It is possible some cognitive therapy would also help, but psychosocial would be the best choice.

A client who is HIV positive is seeing a social worker in private practice for depression. The client names several partners with whom he has had sexual relations. The social worker encourages him to notify his partners of this, but the client refuses. In a state where no reporting is required legally to the health department or to the at risk victims, what should the social worker do first?A) call the public health department anyway to find out what can be done to protect the partnersB) try to get written permission from client to notify the named personsC) notify the named persons without the clients permissionD) give the client the name of the health department and ask him tolet them know

Answer is A. We should report it to them if the client is refusing to take action to protect those involved. You could try to get permission but this is generally not the role of the social worker. **

Family patterns are changing. When a couple experiences a divorce and the family is forced to reconstitute what can be expected to happen? A) family roles will change B) a change of traditions and culture will occur C) the family will seek counseling D) the parents in the family will set more clear rules

Answer is A. When a family is reconstituted there is a change in structure and sometimes the functions of the way members behave or respond, therefore a change in roles occurs.

Which of the following could present a barrier to client self determination? A) broker B) advocacy C) systems interventions D) education

Answer is A. broker is something you do and arrange for the client.

A male client is seeing a social worker for anger management. He is progressing in treatment as he has not had any anger or violent outbursts in over a month. During one of the sessions he admits to the social worker that he fears he may be come so angry with his wife that one day he will kill her. What should the social worker do first? A) explore his fears about losing control B) warn the wife of his threat C) tell him you must warn his wife of his potential threat D) clarify what he would do if he lost control

Answer is A. help him to explore his fears first. Once this is done, another course of action may be warranted.

Cognitive restructuring therapy is an appropriate technique when a client has. A) bipolar disorder B) dysthymia C) dementia D) schizophrenia

Answer is B, dysthymia. In this milder form of depression CBT is often used.

In a session with a client the following communication occurs between the client and the social worker. What is this most likely called? Client: I am so unhappy in this relationship. Social Worker: You feel this relationship is the cause of your unhappiness? A. confrontation B. interpretation C. reflection D. paraphrasing

Answer is B, interpretation. The two best choices are B and C. But it is easy to get B, C and D confused. The key is the word, you feel and that the social worker is not just paraphrasing as they are clarifying and interpreting (not just restating in a different way) what the client feels. You are interpreting it. Client states: I am so unhappy. Social Worker Response: In paraphrasing, it would be: The circumstances within your relationship make you unhappy. In reflection: The relationship makes you unhappy. In interpretation: Your relationship is the cause of your unhappiness. In confrontation: You have done nothing to make changes in your relationship, what else could you do to take responsibility to make a difference?

Two agency directors want to meet with another agency. Their goal is to make sure services are needed in the community and once identified can be addressed. This phase of the planning process is called: A) implementation B) needs assessment C) preparation D) monitoring evaluation

Answer is B, needs assessment.

From a psychosexual perspective, where does guilt stem from? A) the ego B) the super-ego C) the id D) the conscience

Answer is B, the super-ego.

Which diagnosis best fits a man whose arm will not function but the doctor says there's nothing wrong with it. While serving in the war, the man was a soldier who once threw a grenade that killed a group of children. A) malingering B) conversion C) somatic Pain Disorder D) schizophrenia

Answer is B, this is a conversion disorder.

A social worker is working with a client that suffers from developmental difficulties who cannot come up with goals for treatment. The social worker has been trying to get the client to work with her on mutually negotiated goals for the last four sessions. What should the social worker do? A) the social worker should set the goals B) the social worker should suggest goals to the client C) use an approach that doesn't have goals D) leave the determination of goals for a later session

Answer is B. A is possible but the social worker should suggest the goals so the client has one last chance to participate before setting them for the client. Mutually negotiated when ever possible....

A social worker is working with a client in session. The client tells the social worker that her 13-year-old daughter was raped last week. The mother is upset and angry regarding what has happened to her child. What should the social worker do first: A. Help the Mom focus on the needs of her child B. Focus directly on the needs of the child C. Offer to support the child D. Acknowledge her anger and help the Mom to focus on processing her own emotions about this event

Answer is B. Acknowledge the client's anger and help her address the feelings. Once done she can help her daughter. If both were in the session (mother and daughter that was raped) always help the victim first and later in the session you would address the feelings of the Mom.

A social worker has been seeing a female client for several months. Her boyfriend calls you very upset and angry saying you are ruining their relationship. What should the social worker do? A) tell him to talk to the client about a joint session B) refuse to talk to him C) let him finish what he has to say and simply end the call D) acknowledge that his girlfriend is working on the relationship

Answer is B. Answer is between B and C. Generally we do not talk to anyone about a client, so B is the best choice. C is a possibility but it is best not to refuse to talk with him at all.

A social worker has recently been assigned a new child on her case load. The child is 2 years old and lives in what could be considered a high-risk family situation. Which of the following is most likely a sign of child abuse? A) a broken ankle B) a spiral-twisted shoulder C) bruises on both legs D) a mild concussion

Answer is B. Any type of a twisting injury is unusual in a child that age. The other types of injuries are more possibly related to an accident.

A social worker is working with a 19 year old client who states she was sexually abused by her father for eight years. She states that the abuse began when she was 10 years old. She also has two younger sisters ages 16 and 11 that still live at home. When the social worker asks whether her father might also be molesting her sisters, she changes the subject. In this situation, the social worker should: A) report child abuse, since the abuse of your client took place while she was under 18. B) report the social worker's suspicions that the client's sisters are being abuse C) get more information for your client so that you can determine whether a child abuse report is necessary D) maintain client confidentially and do not file a report

Answer is B. As a mandatory reporter we are expected to REPORT suspected cases of physical/sexual abuse. The reason C. is not the answer is that the report is necessary. You could get more information to substantiate the report, but it will need to be made.

From the choices below, which type of therapy might work best for Asian families? A) psychodynamic B) problem solving C) structural D) strategic

Answer is B. Asian families tend to be very private and might not feel comfortable with pyschodynamic or strategic family therapy. Two best choices are B and C, with B, being the better choice.

A social worker is conducting a domestic violence group of male abusers. One of the group members with a violent past shares that he has started dating again and describes the woman he is now seeing. Another group member figures out that he is the brother of the woman the group member is now dating. The group member states "Hey that sounds just like my sister." What should the social worker do? A) remind the clients that all information shared in the group is confidential B) in the group setting ask the client to discuss how he feels and what he plans to do about the information that he has just received C) discontinue the discussion and change the topic D) encourage the brother of the client to ask the other group member to consider dating someone else

Answer is B. B is the best choice, followed by A. What is shared in the group is not legally confidential but represents a promise among members. If the client feels, however, his sister could be at risk, the client will take action. It is better to bring it up in the group and see how the person responds.

What is the best way for a social worker to measure intervention success with a client that suffers from a social phobia? A) questionnaire B) pretest posttest C) ask client about success D) ask family members about progress

Answer is B. Best way is to see how she did before and after measuring what changes occurred, this is the pretest posttest.

The social worker is seeing a coherent and mentally competent elderly woman who had been placed in a nursing home by her daughter. The woman is now requesting to be discharged from the nursing home and wants to go home to live with her son. The daughter tells the nursing home that the son has a drinking problem and should not be permitted to care for their mother. The son insists his mother be discharged and that her wishes to live with him need to be honored. The social worker should: A) call the daughter to inform her when the son is coming to have her discharged B) as her client is mentally competent, advocate for her clients wishes to go and live with her son C) call a meeting between the son and the daughter and the nursing home staff D) respect the daughter's wishes for she is listed on the mothers report as the immediate family involved and the one who set up the placement

Answer is B. But a meeting would be the second choice if you could get them together. The wishes of the client would be followed regardless of what the daughter says. **

A family was reported for suspected child abuse. The social worker is going to the family home to complete an intake regarding the suspected abuse. When the social worker arrives at the home she notices that the two children ages 5 and 3 have bruises on them and seem to be afraid of their mother. What is the FIRST thing the social worker should do? A) remove the children from the home and place in emergency foster care B) complete a thorough assessment C) suggest the family seek therapy D) get information from neighbors about the home situation to support your concerns

Answer is B. Complete an assessment first before you take any action.

In communication theory, the person receiving and interpreting the information is. A) encoding B) decoding C) forwarding D) backing

Answer is B. Decoding is often used in communications theory

The family of a 70-year-old man brought him in for assessment. His symptoms include sleeplessness, forgetfulness, and labile mood. The Dr. says the man has Alzheimer's disease. The social worker should than screen for: A) anxiety disorder B) depression C) schizophrenia D) brain disorder

Answer is B. Depression and dementia often go hand-in-hand and it is hard to tell the difference. This is very true in the early stages.

During the initial session with a social worker, the client's level of motivation could be assessed by: A) attempting to establish therapeutic relationship B) exploring the reason the client has entered treatment C) observing clients body language D) ask about usual means of coping

Answer is B. EXPLORE why the client has come to treatment. This will set the tone. Once done, you can establish the relationship to gain further cooperation and trust.

The social worker is working with a client that for the most part manages his own activities of daily living but suffers from a learning impediment that stops him from developing his own goals for treatment. What should the social worker do? A) put off goals for later B) work with the client and suggest some goals C) start the treatment without setting goals D) ask the parents what goals they would like to see

Answer is B. Each session/intervention has to have goals and objectives. We always want them to be mutually negotiated so by suggesting them and working with the client, the client gets a chance to participate.

A couple is seeing the social worker for communication problems. During the interview the wife sticks her tongue out at her husband while he is talking and makes some derogatory remarks about what he has said. What should the social worker do first? A) refer the wife for individual counseling B) have husband respond to wife's gesture and remark in session C) address the wife and her behavior in session D) suggest "I" statements

Answer is B. Empower the client and if possible facilitate communication. If that does not work, the social worker would address it. **

A 30-year-old female client is being seen by a social worker under a court order for being convicted of physically abusing her son. She states that she is sorry and wants to get her son back. After the second session the client states she feels he does not need treatment anymore and is considering not returning next week. What should the social worker do? A) remind her she has no choice and must return since this is a court ordered treatment B) explore with her in the session the consequences if she decides to make this decision C) acknowledge the rights of the client for declining the treatment D) inform the court about what happened during the therapy

Answer is B. Explore the consequences of her decision. The social worker will have to report this to the court and what will this mean for her and regaining her parental rights?

A social worker is working with a client and her daughter in session. The 13-year-old daughter was raped last week. The mother is really upset and angry regarding what has happened to her child. What should the social worker do first: A. Help the Mom focus on the needs of her child B. Focus directly on the needs of the child C. Offer to support the Mom D. First help the Mom to focus on processing her own emotions about this event

Answer is B. First help the victim and later you can address the feelings of the Mom.

A client with borderline personality disorder soon becomes very critical of her social worker. What does this mean? A) she has lost interest in the social worker B) she feels very committed to the social worker C) she feels violated by the social worker D) she is trying to assist the social worker

Answer is B. From the choices listed the best answer is B. When a client with Borderline Personality Disorder is committed to the social worker it is often expressed in critical, overly-dramatic and difficult to balance and set boundary type relationships. This is why the relationships they have are so intense.

A social worker has been counseling a couple once a week for several sessions. After the session has ended and the husband leaves the room, the wife of the couple asks the social worker to also see her alone once a week for individual therapy. What is the best ethical course of action for the social worker? A) explain to the client this is unethical and discontinue seeing the couple B) recommend that the woman see another social worker for individual counseling C) explain this has to be a couple decision and bring it up at the next session and let the couple decide D) ask the couple to decide if they want individual of couples counseling

Answer is B. Generally we do not see a couple for both couples and individual as it creates a conflict of interest. It is better to get someone else to do the individual counseling. **

A social worker works in an emergency room. She works very long 12 hours shifts. She has a co-worker who is constantly late to relieve her from her shift at work. What should the social worker do first? A) report the co-worker to the supervisor B) confront the social worker C) announce it in staff meeting D) leave at your scheduled time

Answer is B. Generally, we confront before we refer. I would say confront the social worker about his or her behavior first. We always do this in an ethical situation. This is bordering on ethical...not really but I say if a client is harmed because of lateness...maybe. Anyway, I would go with B as the correct response. If that does not work go with A and report to the supervisor. Allow the social worker to take corrective action first. The two best choices are A and B. Get it down to two choices and make your best educated guess.

A social worker is running a substance recovery group. She appoints a member to lead the discussions in the group and provide a type of peer-support. The leader the worker appoints spends too much time talking about his own bad behavior. The leaser does not seem to refer this back to the group and process it accordingly so it is beneficial for the whole group. What should the social worker do? A) discuss what is problematic about the leader's style with the group B) initiate a group discussion related to the goals and objectives of the group C) tell the assigned leader he needs to focus on the agenda D) ask for a co-facilitator to improve the communication

Answer is B. Get the group to re-process the goals and objectives. It would not be appropriate to challenge the leader without first trying the group process.

A client is considered mentally competent and is released from a psychiatric hospital. She was diagnosed with Schizophrenia in remission. The social worker is concerned as she lives alone and she does not want to swallow pills and has to take medications daily. The social worker is concerned she will not take her medications. What should the social worker do to facilitate the discharge? A) talk with the client about why she does not want to take medications B) help to develop a plan to help her take the medications and to report problems C) tell her if she does not take the medications as prescribed she will be back in the hospital D) ask her what she plans to do to make sure she takes her medication

Answer is B. Help the client to have a plan and problem solve when she is having trouble taking them

A client has recently been raped and beaten by her attacker. She wants to go to group therapy but her insurance company will not pay for it. What should the social worker do? A) call the insurance company to advocate for the client B) refer her to a rape support group that has no fee C) explain to the client she cannot have group treatment under her policy D) tell her to call her insurance company and request it again

Answer is B. If a service is available and she can go there I would refer her there. It would not do much good for the social worker to call the insurance office of the client as this would be the role of the client.

What is the best course of action for a social worker to take when it is decided as part of utilization review (UR) that a client needs to be discharged because of insurance reimbursement denial. The social worker believes the client is still in need of outpatient treatment for PTSD related to a violent rape. A) encourage UR to advocate for the client B) tell UR it is inappropriate to discharge this client C) discuss payment options with the client D) refer the patient to another agency

Answer is B. If the social worker feels this person should not be discharged it is appropriate to advocate on the client's behalf.

A worker just found out that one of his colleagues at a different agency has been assigned to his former client who has recently been transferred there. The first social worker has information on his former client that he gained while the client was in therapy with her. The social worker wants to share this information with the other social worker as she thinks it would be helpful. What should the social worker do to get the information to his colleague? A) call the colleague and supply the information regarding his former client B) not mention the situation unless a release is requested by the client C) call the other agency, ask to speak with the client to obtain consent to call his present worker to offer any insight if needed D) it is unethical to give 3rd party information on a client

Answer is B. If you have consent you can do it, but only if you have consent and the client requests it. I would say B, as you do not have a release and the client has not requested it.

A social worker in an agency is visited by a Native American Grandfather. He wants to talk with the social worker regarding the fact that his grandchildren have no plan for permanency and have been in the system for over a year. You acknowledge his concern and begin to talk about options for permanency. He quickly interrupts and starts talking about his grandfather being killed by the American soldiers' years ago. What is the best course of action for the social worker to take? A) redirect the client back on task B) attentively listen to his story C) directly tell him to that is not important right now D) tell him if he wants his grandchildren back to stay on the topic

Answer is B. In the Native American culture respect for the elder is critical. Once you have allowed him to speak his mind you can redirect him back to the task. Best choices are either A or B. This is the best way to gain his rapport and trust.

Psychodynamic theory assumes that change is primarily produced by: A) cognitive restructuring B) insight C) expression of feelings D) future events

Answer is B. Insight is the key.

How does behavioral therapy and cognitive therapy work together? A) cognitive Behavioral therapy focuses on the behavior first B) this therapy recognizes the thought that leads to the behavior C) this model recognizes how new behaviors cause new perceptions D) biological drives always cause the behavior change

Answer is B. It is the combination of the thought leading to the behavior.

A social worker is terminating with a client who is 68 years-old. The age of the client is similar to the age of the social worker. At the last session, the client says "I think we would be good friends." What should the social worker do? A. Agree to meet the client for lunch B. Explain to the client that our professional code of ethics would not allow a friendship to develop C. Tell the client it would be okay to become friends after the sessions are over D. Tell the client it is unethical to ask that the social worker do that

Answer is B. Once a client always a client. It is not unethical for the client to ask, as many clients do not know the social work professional code of conduct, but the social worker should.

The primary influence of the development of self image among children of minority status is from A) the media B) the family C) treatment by others D) experiences with peers

Answer is B. Other ways may influence but family is the key.

A social worker who specializes in working with children that have Aspergers syndrome is contacted by a parent. The social worker schedules an evaluation the next week. Two days later, the preschool teacher makes a referral on the same child. What is the social worker's mostappropriate response? A) explain to the parents why you need a release. B) recommend that the teacher contact the parents C) obtain as much additional information as you can from the teacher while you have her on the phone D) explain to the teacher you cannot accept the referral

Answer is B. Recommend that the teacher contact the parents about the need for a possible referral.

A social worker is working with a couple in the process of divorcing. The couple has two young children and both parents want shared custody. The social worker is doing parent mediation for the couple. In the first session the couple report they are sharing custody of the children but live separately. In the interview the social worker discovers that the mother has custody but the father has unsupervised visitation every other weekend with his children. Upon discussion with the couple it is revealed that the father likes to look at pornography, and has shown his genitals to the children. In addition, he would put his children to sleep in bed with him. What should the social worker do first? A) explore the reason for the father's behavior B) report the case of potential child abuse C) acknowledge the fathers concern to stay close to the children D) ask for the courts to have this unsupervised visitation be reevaluated

Answer is B. Report the case as this clearly falls under mandatory reporting.

A deaf client comes into the social work office with an interpreter. When the social worker is asking questions to clarify what the client needs who should the social worker look at while asking questions? A) the interpreter only B) the client only C) look at neither as not to offend D) eye contact is not an important factor for interpretation of what is said

Answer is B. Start with what it is not. It is not choice D. Eye contact is very important for recognizing understanding. It is not C; in this case, it does not fit. Although the interpreter is speaking it is the client who is the focus of attention. So answer is B. Always look at the client.

A social worker is seeing a client for the third session. The client surprises the social worker by moving into the house next door to her. What should the social worker do first? A) the social worker should move B) talk with the client and explain boundaries C) terminate services with the client D) refer the client to another social worker

Answer is B. Talk with the client and explain boundaries. Once this is done, you may need to refer the client to another social worker because of the potential for a dual relationship

A social worker was abruptly terminated from an agency without being able to terminate with clients. What should the social worker do? A) talk to the agency director B) talk to her supervisor and inform her that it is ethically wrong not to be allowed to terminate with your clients C) inform clients by mail that you will no longer be working at the agency. D) leave the agency as directed as you are aware that no follow-up will be provided for the clients

Answer is B. Talk with the supervisor as we always try to terminate with clients and just not leave abruptly.

What is the role of a Board of Directors? A) develop and implement goals B. make community aware of the agency and carry out the mission statement C) hire and fire staff D) develop a temp-plate for service delivery

Answer is B. The Board works with the mission statement and the director carries out the goals and objectives.

When a mother is working with her child, which one of the following child behaviors would most likely be linked to attachment theory and a problem with attachment? A) child coloring instead of playing dolls with Mom, B) child seeking comfort from social worker instead of Mom when hurts himself C) child looking out of window when mom is talking to social worker D) child asking Mom repeatedly to leave the session and go home

Answer is B. The child is seeking comfort from the social worker instead of the mother.

A social worker's responsibility to work and practice within their competence falls on the: A) agency B) social worker C) supervisor D) administration of the agency

Answer is B. The code of ethics is clear that the responsibility to act in a competent way falls upon the social worker. If the social worker is working under the auspice of a clinical supervisor the vicarious liability may rest with the supervisor, but the social worker is ultimately responsible for his or her own behavior.

A social worker has learns that a father becomes quite angry when his two-year-old son soils or wets his pants. The father's usual response to this behavior is to severely spank the child. The father's behavior most likely reflects: A) a need to explore his relationship with the child B) a distorted perception of developmental milestones C) a need to develop new ways to cope with stress D) displacement of his anger toward his wife

Answer is B. The father's behavior shows he does not understand normal human growth and development.

A social worker has designed a survey for respondents to take that have used a service she is providing at her agency. The worker wants to get a representative number of respondents utilizing the service to participate. Which method of sampling will help the social worker get the most representative sample of client responses? A) a convenience sample B) a random sample C) a cluster sample D) a non-probability sample

Answer is B. The random sample is a probability sample that ensures that "each and every individual has an equal chance of being selected." A cluster sample is a non-probability sample and so is the convenience sample. The one that has the strictest rules but is the most representative is the random sample.

A social worker is working with a client that states he is getting married and invites the social worker to come to the wedding. The therapy sessions are ending and the client has progressed well. What should the social worker do first? A) Tell the client it would be unethical to attend B) Explore why he wants the social worker to attend and use this as part of the discussion regarding termination C) Do nothing and tell the client you need time to decide whether to attend D) Accept the invitation graciously

Answer is B. The social worker can first explore/inquire why and begin discussion about termination limits at this time. It would not be unethical to attend the wedding.

A social worker is leading an all female group. During the second session the social worker asks a member who had been silent if she wanted to share and the member responded abruptly with "leave me alone" and walked out of the group. What should the social worker do first? A) ask another group member to invite her back B) explain to the group that everyone is expected to share experiences C) tell the group she is not ready to share at this time D) stop the group and bring her back

Answer is B. The social worker is clarifying/explaining the rules and expectations of the group. This should be done first.

A cognitive therapist repeats what a client has said repeating the phrase "I am nothing." What is this an example of: A) venting B) is assisting the client in a self learning process C) is using a none directive approach to problem solving D) is teaching the client what he is teaching others about him

Answer is B. The social worker is helping the client to recognize negative statements and thereby facilitate learning.

A couple has been married for five years seeks counseling. The wife convinces the husband to come because he is having a problem with impotence and intimacy problems. After several sessions the couple appears to be doing much better and report that the problem with impotence has been resolved. At that point, the wife reveals in the session that she has decided to end the marriage and is leaving. What should the social worker suspect? A) the wife was having an affair B) the couple had other problems that were not yet addressed C) the wife wanted out of the marriage from the start D) the husband had more problems than the one stated in the session

Answer is B. There were probably other concerns and this one was at the forefront of the others, also known as the presenting problem. Once this one was addressed the others became more obvious and there was little desire to address them.

A social worker conducting research best controls for threats of internal validity by using A) participation observation B) randomization techniques C) posttest only D) longitudinal studies

Answer is B. This is a difficult question. Let us start with what it is not. It is not A, D and or C. These are designs. In addition, B will help with reliability but not validity but it is the only one related to it. Must be B...but this does not really make sense yet it is the only one that fits.

A social worker is using the technique of the "empty chair" with a client that who is grieving the loss of his partner of 20 years. The partner died of cancer and was very ill for the last six months after the diagnosis. The client feels that so many issues were never addressed with the now deceased partner and now it is too late. Which choice below best describes use of the empty chair method? A) ask the client to sit in the empty chair B) pretend that his partner is in the empty chair and respond to address unfinished business C) the social worker will sit in the empty chair and answer for the partner D) the social worker and the client will engage in role play with each other regarding the death of the loved one

Answer is B. This is an example of how the empty chair is used. Although it is a type of psychodrama or role play D is not the best answer.

A woman in treatment with a social worker comments that whenever her adolescent son becomes angry, she feels as though she is a failure as a parent. The social worker comments that many adolescents can act that way and being angry at times is a normal phase of development. The social worker's technique is known as: A) clarification B) interpretation C) confrontation D) universalization

Answer is B. This is an example of interpretation.

When a therapist goes over what was discussed in family therapy, and goes over what was agreed to be worked on for the next session, this is called: A) focusing B) summarizing C) rephrasing D) clarifying

Answer is B. This is close as it could be either B or D, but B is stronger.

During an in-service workshop the facilitator ask clients to role play. This is done to the help the participants learn: A) better understanding of communication responses B) experiential training methods C) tools used to enhance objectivity and responses D) an understanding of emotive responses

Answer is B. This is experiential, where the participant actually experiences what is happening.

Two children were neglected by their parents and left in their bed all day watching television. They were rarely held and got little attention from the parents unless they had to respond. What is the most likely diagnosis that could result? A) pervasive mental disorders B) reactive attachment disorder C) attention deficit disorder D) separation anxiety

Answer is B. This is most likely going to result in reactive attachment disorder where the child does not bond to the parent.

An attractive client says to the social worker "I feel so ugly," Which reflective statement by the social worker acknowledges and indicates acceptance of what the client is currently feeling? A) here look in this mirror and tell me that you are not attractive B) right now you don't feel very attractive C) you are very critical of your self today D) you are not happy

Answer is B. This is the response that starts where the client is.

A foster care agency was trying to place a child in a foster home and there is only one foster home available. A report is made that the social worker assigned to the case for placement of the child previously had a sexual relationship with the foster mother in the only available home. The relationship is over now and at the time there were no children placed in the home. The social worker supervisee discusses this with his supervisor and states it should not be an issue as the relationship is over and there were no children in the home at the time. What should the supervisor do first? A) report the Social Worker's action to the appropriate authorities B) release the child to a different agency, which does have a home available C) conduct an internal investigation about what actually happened D) placed the child in the same home as planned

Answer is B. This is what needs to be done first. The next thing is to report the social worker as this could be a violation of the Code of Ethics and an internal investigation may be warranted. Dual relationships with fellow colleagues are considered conflicts of interest.

In an initial session, a client reports how much his life is a mess and mentions several family issues, kids, financial, marital, and his problematic parents. The technique most likely to help the social worker facilitate problem management for the client is: A) active listening B) partializing C) empathizing D) advising

Answer is B. This requires and answer that MOST supports problem management, of the choices the technique that most supports problem management is partialization.

Can a social work supervisor be held responsible for after the suicide of a supervisee's client. IN supervision the supervisor had implemented a system of only reviewing three charts per supervision session and the dead client's chart was one that wasn't looked at. A) no the supervisor is not responsible as she did not review the chart B) yes the supervisor is responsible and has vicarious liability C) no the supervisor is not responsible and no further action was needed on the supervisor's part D) yes, the supervisor is responsible as she did not arrange a time to review every record accordingly

Answer is B. Vicarious liability is expected as the supervisor is considered responsible for the entire case load whether they review the chart or not.

A social worker is trying to help and adult child decide whether to place his elderly parent in a nursing home. The physician has recommended placement in a nursing home. What should the social worker do first? A) work with the family member to admit the client as requested B) evaluate the client's needs and help to provide options to help the family member decide C) tour the facility with the family member D) educate the family member to the services that can be provided at the nursing home

Answer is B. We evaluate before we make a recommendation. If you are trying to help the client to decide empower them by helping to evaluate and provide options.

A social worker is working with a four-year-old child and suspects that there may be an attachment problem. What behavior would be most consistent with a child that has attachment issues: A) when Mom enters a room the child continues to play alone or color instead of playing with Mom B) when child falls down, the child runs to social worker instead of Mom C) child goes to look out of the window while Mom talks to social worker D) a child picks a bright colored toy as an activity over reading a book with Mom

Answer is B. When a child is hurt he/she will almost always seek the mother for comfort this is a very basic part of parental child bonding.

A social worker is running a group. What is the best sign that individuals participating in the group are ready to terminate? A) the participants become involved in various other outside groups B)frequent absences C)the participants become quiet and no longer speak in group D)the participants make connections with others within the group

Answer is B. When a member starts to miss group sessions it becomes clear that he/she know longer finds the group helpful.

An individual complains he is unhappy with his life and who he has become. The social worker correctly explains his symptoms as ego-dystonic. These terms are correctly defined as symptoms can be "Ego dystonic" when ___________ and ego-systonic when _________________. A) symptoms are not alien to the person and are a part of who the person is; driven by the id and they have little control B) symptoms are inconsistent with a person's sense of self and unwanted; whereas in ego-systonic symptoms are not alien to the person and are a part of who the person is. C) symptoms are driven by the ego and self-regulated; whereas in ego-systonic the symptoms are inconsistent with a person's sense of self and unwanted. D) symptoms are not alien to the person and are a part of who the person is; whereas in ego-systonic the symptoms are inconsistent with a person's sense of self and unwanted.

Answer is B. When it is ego-dystonic there is desire for change as it is not acceptable to the person; whereas in ego-systonic it is acceptable and the person accepts it as natural to him or her. There is also a medical term known as ego-syntonic, this term is generally used in the same way as ego-systonic and the person is comfortable with his or her thoughts and perceptions. Therefore, in ego-dystonic (but no term such as ego-dyntonic) the individual feels his thoughts or feelings are not acceptable or repugnant. Definition: Ego-alien is the same thing as ego-dystonic...another name is ego-tension or bad tension, sometimes related to homosexuality but to use this term the individual must report that they feel repugnant of what they are experiencing.

The parents of a 15-year-old bring their teenager to see a social worker and state that they are concerned as their teenager is withdrawn and acting very strange. When the social worker meets privately with the teen, the teen begins to cry and says she wants what she says to be confidential or she will not speak to the social worker. What should the social worker do first: A. Agree not to tell the parents so the teenager will talk B. Work with the teenager to help her tell her parents of her concerns and set up a joint appointment C. Report exactly what she says to the parents, as it is the law D. Follow an ethical code in your response

Answer is B. You can explain to the teenager what you must do and explore the options she has. You can talk with the parents together and see if she is agreeable to that. The parents can request the information and when they do, the social worker will have to release the record but that does not seem relevant here, and follow our code of ethics is important but there is also a legal aspect here that must be addressed.

An elderly client keeps repeating the same thing over and over to the social worker. The social worker can feel the frustration rising as the client does not seem to hear what she is saying. The worker becomes frustrated and says, "how can I help you when you simply do not listen." What should the social worker be careful of? A) over-identification B) dependence on the client C) counter-transference D) job stress

Answer is C, counter-transference. She is becoming frustrated with the client and this can impact the treatment she is trying to provide.

An administrator in charge of a casework based agency's primary goal should be: A) provide the community with an available and affordable casework staff B) comforting the client C) to establish the most cost effective, cost efficient casework possible D) help clients explore past issues

Answer is C.

Which is the procedure for evaluating staff performance? A) systems theory B) employee evaluation C) performance Appraisal D) program evaluation

Answer is C.

A teenage girl goes to her high school social worker to ask for help in arranging an abortion. She is concerned she cannot afford and abortion and her boyfriend will not help her pay for it. The social worker should first? A) call the teen's parent's against her wishes to inform them of the situation B) call a local abortion clinic to find out price information C) refer the teen for counseling at the health department's family planning clinic D) fulfill the teen's wishes.

Answer is C. Key Words: A) call parents B) call clinic for a price C) refer teen D) fulfill wishes Only a Genie can fulfill wishes. ;) This means it has to be one of the other three and refer teen is next best choice. **

What can parents expect from a healthy six year old child in terms of what the child is striving to achieve? A) self-assertion B) self-trust C) self-determination D) self-efficacy

Answer is C. The term self-determination is most likely to occur (Erikson) between ages 3-6 and involves determining a sense of initiative, purpose and direction. The term self-trust relates most closely to Erikson Trust vs. Mistrust that occurs at approximately 0-12 to 18 months. The term self-assertion is related to Erikson's writing and generally occurs between 18 months and three years. The term self-efficacy involves our belief in our ability to succeed in specific situations. The teens and the adults approach to self-efficacy can play a major role in how goals and objectives are approached. The concept of self-efficacy lies at the center Bandura's social cognitive theory.

An 8 year-old girl who was recently adopted was hoarding food in her room. The new adoptive parents are very concerned and say they always make food available and snacks when needed. They do not understand this behavior and are not sure what to do with it. In the assessment phase what should the social worker consider as the most likely cause of this behavior? A) a sign of sexual abuse as a toddler B) a sign of a future eating disorder C) normal behavior that should be watched D) a sign of witnessing domestic violence in her past

Answer is C. Although the other three are possible C (normal behavior) is the one that is MOST LIKELY the cause. Hiding food is not uncommon in children who have been neglected, abandoned or not fed regularly. Some children may hide food in drawers, under beds or even in clothes and this can be related to childhood insecurities or neglect. The best way to address is to have snacks available and constantly reassure children you will take care of them. Provide regular mealtimes so that a child learns to trust that he will be cared for. Provide reassurance and this behavior may lessen when a child feels stable and cared for so do not punish a child for hiding food. Provide reassurance.

A social worker is working with a couple in therapy. The wife states that she had an affair because her husband is a truck driver and he was gone all the time. During the session, the wife states she does not want the marriage to end. She than suggests that the she be allowed to go on the road with him from time to time. The social worker discusses this as an option and both agree to do this. In this scenario the social worker is: A) manipulating the therapeutic session B) the social worker is placing her values on the couple C) encouraging cooperative decision making D) empowering the husband to have more control

Answer is C. Based on the choices provided, the best response is C, as the social worker is working with the couple to get them to take action steps to deal with the problem.

Cognitive therapy is best for treating A)problematic behaviorsB)anxiety related behaviorsC) dysthymiaD) somatoform disorder

Answer is C. Cognitive therapy is used with patients that suffer from dysthymia. It can also be used with problematic behaviors and anxiety related behaviors. If it said, cognitive behavior therapy, I would have selected anxiety related behaviors. Look carefully at the answers. **

A client is seeing a social worker and frequently expresses frustration over the fact that the school is always calling her work to come and get her son from school due to behavior problems. The client invites the social worker to her child's school meeting to discuss behavior concerns. The mother asks the social worker not to mention anything about her own mental health problems to the school team. What is the social worker's best response? A) attend the meeting and respect the client's right to confidentiality B) attend the meeting and reveal the mother's diagnosis because you believe it will help in understanding the child's problems C) decline the meeting based on boundary issues D) tell the client you cannot attend the meeting if you cannot be honest about what you know

Answer is C. Decline the meeting as it appears that boundary issues could be a problem.

A couple agrees to see a counselor for marriage therapy due to intimacy problems. Prior to the session, the wife calls you and reports she is having an affair. What should the social worker do next? A) agree to see her individually B) explain to her individually how detrimental the affair could be C) encourage the client to bring up the affair in the session with her husband D) refer to another counselor

Answer is C. Discussion needs to be done as a couple, this is critical if couple counseling is to exist.

Which is considered the best way to evaluate the effectiveness of behavioral interventions in a community mental health clinic? A) write to client to see how he/she is doing B) do a 6 month follow up phone call C) do random selection of charts to review outcomes D) do a systematic review of all charts

Answer is C. Do a random review to check for outcomes.

A client has been approved through her managed care company for 15 sessions. The client is at the 14th session and the SW believes it will take 20-30 sessions to reach goals. What is the first step the social worker should take? A) offer sliding fee scale to continue seeing the client B) terminate early C) file an appeal with the managed care company D) address the situation with the client

Answer is C. File and appeal for an extension.

A client has asked to see his medical record. The social worker feels it is not in the client's best interest to review the record as he is not thinking clearly at the time of his request. What should the social worker do first? A) explain to him he cannot have the information now because of his condition B) tell the client to come back later and you will help him C) clarify with him why he wants the information D) explain to him the procedure for getting the information from his medical record

Answer is C. First clarify why he wants the information, once done you need to explain the procedure for getting the information. C and D are the best choices.

A young social worker manager is feeling frustrated as workers are coming to work late and complaining about the job related duties they have been assigned. How can the social work manager most effectively minimize the current situation? A) meet with Board and ask them to explore worker family leave policies B) ask administration for raises for the employees to increase motivation C) let each worker know that they will need to stay extra time to make sure work is done D) let the agency director handle it

Answer is C. Hold people accountable...and C comes closet.

A social worker knows an individual through contact with a local business for the last ten years. Through his professional job responsibilities he is court-ordered to do an evaluation on this individual. What is the responsibility of the social worker in response to this request? A) call the court to tell them about dual relationship B) tell the client about the situation and ask his advice C) refuse the case related to a conflict of interest D) tell the job and let them decide how to handle situation

Answer is C. If a case presents a conflict of interest the social worker should refuse to take or participate in the case and render a professional opinion.

A social worker has recently been assigned a new case in Hospice care. The client has terminal cancer and states she does not want to go on taking the treatments to prolong her life as they make her so sick. Her family, especially her daughter who cares for her at home, is against this and has openly stated to the social worker she will not support this decision. What should the social worker do? A. get a second opinion from the medical staff B. support her decision and inform the family C. offer to provide family therapy to help the client to understand the daughter's reasoning for non-support D. refer her to her church clergy to provide the counseling

Answer is C. In this case, you are offering to the client to help her work with the family and her. The family will need to carry out the plan she has decided so their support is crucial. If you do not work with the family they will not support the clients decision. In this case, best practice "should" involve her and the family. Simply supporting her decision may not be enough since her daughter is refusing to honor it.

A couple that a social worker saw for marriage counseling in the past sends an invitation to the social worker to join them for their 20th anniversary. They are having a large party and credit the social worker for helping to keep them together. They state they would be honored if the social worker came as one of their guests and request and RSVP. What should the social worker do? A) decline the invitation in writing B) accept the invitation in writing C) call host to explain why the decline D) accept the invitation but only stay for a few minutes

Answer is C. It is best to call and explain what the social worker will not be there but you can also thank them for the invitation.

A social worker meets with an elderly client and his caregiver. The elderly client appears dirty, malnourished, and has soiled himself and does not make eye contact with the social worker. What should the social worker do? A) report to case to the proper authorities B) make sure the client gets home health care services C) meet with the elderly client and the caregiver separately D) ask the caregiver when the elderly client ate last

Answer is C. It is important to assess for potential abuse and this needs to be done in separate interviews.

In a managed care system a client's services are most often determined by: A) social worker treating the client B) licensed Psychiatrist C) the clients wishes and preferences D) the administration

Answer is C. It is the client that will generally determine what is covered. Managed care plans are health insurance plans that contract with health care providers and medical facilities to provide care for members at reduced costs. These providers make up the plan's network. How much of your care the plan will pay for depends on the network's rules. Restrictive plans generally cost you less. More flexible plans cost more. There are three types of managed care plans: Health Maintenance Organizations (HMO) usually only pay for care within the network. You choose a primary care doctor who coordinates most of your care. Preferred Provider Organizations (PPO) usually pay more if you get care within the network, but they still pay a portion if you go outside Point of Service (POS) plans let you choose between an HMO and a PPO each time you need care

A social worker has been successful working with a client for three months when client reveals a matter that the social worker has no knowledge about, what should the social worker do: A) refer to another therapist in the agency that specializes in that area B) read current journals that relates to subject matter C) ask the her supervisor about the information and how to handle it D) deal with it best she can

Answer is C. Never work outside of your competence level.

A school social worker is seeing a 10 year old boy who stated he would not participate in a school Christmas holiday play. While discussing this with the student, he states that his family is Jehovah's Witness and becomes angry and refuses to continue talking with the social worker.He does not make eye contact and looks away. What should the social worker do first? A) talk to the teacher about what has happened B) call the parents C) clarify with the boy what is bothering him D) tell the boy he needs to participate in the school activities.

Answer is C. Once you clarify what the problem is, the social worker would than talk with the teacher.

A social worker in the emergency room is assessing a 10 year-old client that is suicidal and has a plan to implement his intention. What should the social worker do first? A) immediately contact the hospital seeking inpatient treatment B) contact the parents about the situation C) explain to the client that inpatient admission is needed and try to get consent D) notify a psychiatrist to complete the assessment

Answer is C. One general principle is always if a client is capable of understanding (and determine if this is the case) try to get consent of a client and make sure they are aware of what consent and inpatient admission means, regardless of age. The bottom line is the social worker is always going to explain the procedure and try to get consent regardless of the age. If a child is 14 or older parental consent may not be needed at all. If under 14 it becomes more difficult and the particular agency policy should be adhered and this usually is focused around the Mental Health Act of 2001-2003. Generally children ages 7-13, child consent can be honored in substance abuse treatment but even with that we still try to get family support and awareness. Once you get consent you can notify the parents for permission. In Canada, there is no age for parental consent if involuntary admission is needed but in the US we generally look at over age 13 especially in cases where the child is a danger and no parent is "available or findable" for example the child is a runaway. **

A social worker is working with a client in therapy. The client states she wants her husband to join the sessions as she feels she needs marital counseling. The social worker does not feel the client is ready yet for couples counseling, as she has not yet addressed many of her own individual problems. What should the social worker do? A) tell the client to invite her husband to the next session B) go along with the client and do as she requests C) explore the client's accomplishments in individual therapy thus far D) validate her feelings of frustration with the individual therapy

Answer is C. Review/explore the accomplishments thus far with her independently to help provide a solid basis for her to make a decision.

A women seeing a social worker talks about her anger in having to care for her mother who is forgetful and difficult to control. She was looking forward towards retirement and now is upset because she has to care for her mother. The worker should help support the client and focus on: A) help the client transition from work to retirement B) accept the transition of generational roles C) understanding gender roles within the family system D) reaching the developmental stage of the individual

Answer is C. Start with what it is not: it is not A, this is about her mother and learning to cope with the changes her mother is experiencing while balancing them with the changes in her own life. Answer A is short-sighted. D is not the answer as this has nothing to do with developmental roles. This leaves B and C. Since she is already taking care of Mom and is upset with her and the new gender role she must assume I would say help her understand first and than the next step is to accept.

A teenage girl goes to her high school social worker to ask for help in arranging an abortion. The social worker is not familiar with the laws regarding this type of a situation. The social worker should first? A) call the teen's parents to inform them of the situation. B) call to find out what the parenting laws require and involve in that state C) refer the teen to an unplanned pregnancy clinic D) fulfill the teen's wishes

Answer is C. The best choice is C. An unplanned pregnancy clinic can help the girl to test to see if she is pregnant and than help her explore options. Once the pregnancy is confirmed they will know the law in that state and what needs to be done. The second best answer is B.

N. a 15-year old male is recovering from a history of repeated drug offenses resulting in drug dependence. His parents are confused by the situation and do not support his efforts toward recovery and blame all the problems in the family on him. The social worker is concerned that N. may relapse. What is family doing by treating him this way? A) enmeshment B) displacement C) scapegoat D) passive Aggression

Answer is C. The family is using N. as a scapegoat. The two best choices are scapegoat and the defense mechanism displacement. In scapegoating the family uses N. as the target or the blame for other failures in the family. Generally in scapegoating there is little or no connection to the cause of the problem or anger experienced. In displacement you direct anger or other emotions from one object to a safer object. For example, the parents may snap at N. because they are upset at the water bill they received. Enmeshment does not fit nor does the defense mechanism passive aggressive.

A social worker in supervision tells her supervisor about a case she is working with. The teenage son in the family is taking the family car without permission. The last time he took the car he brought it back damaged and denied he had done the deed. The parents say they cannot handle them and do not know what else to do. What should the supervisor recommend the supervisee consider for this family? A) refer them to family therapy B) call authorities about the teenager's behavior C) help her problem solve how to help the family D) refer the family to a more experienced social worker

Answer is C. The role of the supervisor is to help the supervisee assess and problem-solve the situation. This should be done first and is the accurate role of the supervisor.

A clinical social worker remarks to a client "could your fear of closed spaces be related to the child abuse that you previously discussed" The social worker is using the technique of. A) clarification B) feedback C) interpretation D) confrontation

Answer is C. The social worker is interpreting what the client has said.

A pregnant social worker is about to go on maternity leave for three months. She is deciding the best plan in her absence. What is the best course of action for the social worker? A) discuss the possible options with clients B) have the same interim social worker fill in for all of your clients C) explain to the clients that she is leaving and introduce them to who will be covering for her D) let the agency set up the proper follow-up in her absence

Answer is C. The social worker should try to make sure that her clients are covered in her absence.

A social worker is pleased with the progress of a session with her client. As the session is getting ready to end, the client brings up a new significant issue that she wants to address. What is the client most likely trying to do? A) trying to control the session B) trying to manipulate the session C) seeking continued attention from the social worker D) exhibiting separation anxiety

Answer is C. This is often referred to as the door-knob communication. It generally means the client has found the session rewarding and helpful and does not want it to end. The answer above that best relates to not wanting the session to attend is that the client is seeking continued attention from the social worker.

A social worker is working with a client that is a concert pianist whom you have been seeing for several sessions. The client has a concert coming up and would like for you to attend and meet his family. A. Accept the invitation and attend B. Accept the invitation and attend but leave before the client's family arrives C. Tell the client for ethical reasons you cannot attend and meet his family D. Refuse the invitation and do not explain why

Answer is C. This would be problematic within our code of ethical conduct. It is best to explain this to the client that your relationship with him is a professional one and let him know that although you are honored by the invitation why you cannot attend.

A social worker working for an Employee Assistance Program is referred a client who is having difficulty at work with uncontrollable anger outbursts. Upon the interview, she appears very detached and says she does not like people and that they are the cause of her problems. She reports that several people at work are out to get her and stealing her work items. She screams and yells at them to get her them to stop. It is obvious to the social worker that the woman is having extreme difficulty maintaining her composure and she fears the client may become violent in the session. What should the social worker do first? A) explain the limits of the EAP B) ask the client to leave the room immediately C) acknowledge the client's anger and try to get her to calm down D) run from the room immediately and get help

Answer is C. Try to calm down the client first. Once tried, if this does not work you may want to leave the room or if you have a security system call for help.

A client seeing a social worker is struggling with feelings related to the impending death of his partner who is dying from AIDS. He tells the social worker he is engaging in unprotected sex. What is the client most likely experiencing? A) unprotected grief reaction B) survivor's guilt C) conflicted grief D) adjustment disorder

Answer is C. Two best choices are B and C. Conflicted grief is where the person is internally conflicted and happens particularly when there is a conflicted ambivalent relationship. Survivor's guilt is almost always related to PTSD and is actually one of the symptoms of it. Generally, it is related to the person wondering why they did not survive...and in many cases wishing they did not survive. Survivor's guilt happens AFTER the death.

A social worker is at a social event, a client who has paranoid personality disorder is also there. One of your colleagues is intoxicated and begins to act inappropriately. You observe your client watching you and your colleague. What should the social worker do? A) leave the event B) go talk to the client and explain the circumstances C) get security to control your colleague and help you to get them to leave the party D) separate yourself from your colleague and socialize with other people.

Answer is C. We have a responsibility to assist our colleagues. So answers A and D are not possibilities. We would never do B, as that involves approaching a client and discussing the behavior of a colleague.

A social worker is working with an undocumented immigrant family who is in need of housing, but the family was turned down for benefits by the housing authority because of no Social Security numbers. What should the social worker do first? A) appeal with housing authority as an advocate B) discuss potential problems with the family first, then, with their permission help them file an appeal with housing authority C) attempt to locate alternate housing D) accept the decision by the housing authority

Answer is C. if they have no where to live the first priority would be to find them a place and than help them work within the system.

A school social worker has just completed a session with a teenage girl who has requested assistance from the worker to arrange for an abortion. The school social worker should first: A) call the girl's parents and let them know the situation B) stop the session and call to find out what the state law is involving teen abortions and parental permission C) counsel teen on her options for abortion and/or adoption D) refer the teen to the family planning clinic at the local health department for counseling

Answer is D Key Words in question-social worker is a SCHOOL social worker A) call parents B) call and research law C) counsel teen on options for ADOPTION and abortion, she wants abortion info D) refer for family planning counseling

A client becomes overly dramatic when reporting an event. She cries out repeatedly and stops talking periodically stating that no matter how painful recanting the event is she must go on to finish her account of what happened. Her voice is very dramatic and the presentation appears to be exaggerated. What type of behavior is she most likely exhibiting? A) dependent B) borderline personality behaviors C) passive aggressive D) histrionic

Answer is D, histrionic. This is classic criteria of histrionic behavior.

A family comes to an agency complaining of their child's behavior problems. As a result of information about the parents' interaction provided during the assessment phase, the social worker believes that the marital relationship and the behaviors of the parents could be the primary problem. The social worker should focus on: A) the parents and confronting them with their relationship issues B) the child and ways the parents should react to the behavior C) a discussion of the social worker's perception of the marital relationship D) developing the parent's insight into behavior as a possible reaction to marital stress

Answer is D.

A school social worker is seeing an adolescent who reports having problems at home with his parents. he says his parents will not let him make any independent decisions and are extremely controlling. The boy's parents are threatening to sue the school because they feel their child is not getting the education and treatment that he needs. The social worker notes that there has been progress in the sessions. What should the social worker do: A) Tell school administrators what the adolescent said about his parents being controlling B) Confront the parents directly about their parenting skills C) Ask parents to sign a confidentiality waiver because of a possible law suit D) Discuss with parents your concerns

Answer is D. Start with what it is not. It is not A. We would never do this, there is the whole concept of confidentiality. Answer C, is possible but why would it be related to a potential law suit? B and C are best but confronting the parents is certainly the wrong approach, also, how can you be sure it is their parenting style? Many children believe their parents are controlling as to some degree this is expected.

Retirement is MOST important and satisfying to those that have retired or facing retirement when they: A) live near their children/grandchildren B) are financially secure C) have a home paid for and are mortgage free D) have developed a plan for retirement and activity

Answer is D. All of these choices may be important but most researchers agree planning for retirement, which includes making the choice to retire and making sure you have alternate hobbies, etc. to bring purpose to your life along with good health are of MOST importance. Many individual get their identity from their jobs so re-developing and re-focusing interests are central.

Freud, Erickson and Sullivan have concepts to achieve maturation development. Which is best for describing the human maturational development process? A) adjustment to environmental demands B) conforming to society C) proper conduct D) how well one masters specific life task

Answer is D. All three of these discuss the acquisition of specific life tasks: Freud = psychosexual, Erikson = psychosocial, Harry Stack Sullivan= Heuristic **

A social work director for a non-profit agency is just told the agency will lose about 20% of its funding for one of its most essential programs. What should the social worker director do first? A) mobilize the clients to protest this cut in funds B) establish a budget reflective of the decrease and let other employees know C) start a fundraising drive to replace funding D) meet with staff to help decide which program/services can be cut

Answer is D. Always try to include the staff in decision making when possible. When cuts occur this will make them more likely to feel a part of the change effort. If the manager establishes the budget independently there may be more resistance by staff.

A social worker is working with a couple in therapy. The wife states that she had an affair because her husband is a truck driver and he was gone all the time. During the session, the husband states he loves his wife and does not want the marriage to end. He than suggests that the wife go on the road with him from time to time. The social worker discusses this as an option and both agree to do this. In this scenario the social worker is: A) manipulating the therapeutic session B) the social worker is placing her values on them C) discouraging independent decision making D) empowering the husband to have more control

Answer is D. Based on the choices provided, the best response is D, as the social worker is empowering the husband to be able to take action steps and empowering him to have more control.

A client in the first session with a social worker reports to the worker that he is about to be evicted due to his apartment being messy and unsanitary. As a social worker acting on behalf of the client, the social worker should: A) try to talk the manager out of evicting the client B) help the client to clean up the apartment to avoid the eviction C) help he client to locate a place to live D) discuss the issues surrounding the eviction with the client

Answer is D. Before you can find another place to live, you need to discuss the issues that can prevent it from happening again. Answer D is the best choice.

A social worker is working with a 14 year-old girl and her mother in therapy. The girl is pregnant and wants to keep her baby. The mother thinks she is too young and states she cannot handle a child. The mother reports that her daughter is too immature and still plays with dolls, argues with her little brother and has to be told to do chores. The social worker works in a state where a minor will need parental consent for probable adoption or an abortion. The mother asks the social worker to help her reason with her daughter to give up the child. What should the social worker do? A) as requested help the mother convince the daughter not to have the child B) help the mother accept that her daughter wants to keep her baby C) tell the mother it is her daughter's choice and she needs to accept the decision D) help the mother and daughter explore the options available and the potential outcomes for the choices that are made

Answer is D. Explore and problem solve is the best choice. If this were not there it would be B, to help the mother accept the decision of her daughter.

A couple seeks counseling from a social worker. The couple has been together for ten years. The male partner complains that he does not get along with his in-laws. He states that his wife knows this and continues to want to spend a great deal of time with her family. What should the social worker do to help facilitate the counseling process? A) ask the wife to spend less time with her family B) set up a plan where the wife is spending less time with family C) talk with the husband about his controlling behavior with his wife D) explore what both partners perceive to be the problem between them

Answer is D. Explore the perspective of both individuals to facilitate the counseling process. Once done, from the choices given, A, C or D may follow.

A hospital social worker is working on an interdisciplinary team. She refers a client to an outpatient social worker for depression. After one month, the current therapist phones the hospital social worker explaining that the hospital psychiatrist has not returned her call and the client is experiencing adverse reactions to his medications. What should the social worker do first? A) arrange a meeting with the psychiatrist and the therapist B) discuss the concern with the psychiatrist C ) tell the therapist to contact the psychiatrist herself D) inform the therapist you will get back with her and contact the psychiatrist

Answer is D. First let the therapist know what you plan to do to assist and than assist.

As a social worker, you are called to assess a man that was brought into the hospital by the police for an evaluation. It has been reported that the police officers brought in the man and because of his strange behavior and he was not under arrest. The police were concerned that although he appeared oriented he presented with bizarre behavior. However, upon the social worker's assessment the man is alert but does seem to have some problems with his orientation. He denies having any problems and verbalizes that he does not want professional assistance and is ready to go. The social worker should: A) seek immediate hospitalization B) suggest that he be released C) have police arrest him due to the initial issues D) refer for a psychiatric evaluation

Answer is D. If he is having problems with orientation it is possible that he could be a danger to self, so referral for a psychiatric exam seems most appropriate. If he was oriented and showed no problems with functioning the suggestion that he be released would be most appropriate.

A Gay adolescent's parents want him to seek reparative therapy and want to schedule an appointment with the social worker to provide the treatment and/or give him a referral. The social worker is not familiar with this technique. What should the social worker do? A) agree to see the client and get a social worker skilled in Reparative therapy to coach her B) see the client and ask the client to talk about his feelings in a non-judgmental way C) immediately refer him to someone who knows therapy D) tell the family you are not familiar with that therapy and ask if they would like you to research it for them and give them a call back

Answer is D. If you are not familiar with a therapy you should not promise to use it or refer for its use. First you need to know what the treatment is.

The social worker works for an Employee Assistance Program (EAP). An employee is being seen by the social worker for problems at work. He is a divorced father that has primary custody of his teenage daughter. The daughter has started running away and admits to using illegal drugs such as cocaine. The father that has shared custody with the mother states that he received a call from his ex-wife while he was at work stating he needs to immediately deal with his daughter's behavior. The father left to get his daughter and did not complete a work-related project and is concerned about losing his job. What should the social worker do? A) provide parenting tips for the father B) ask the entire family to come in for counseling C) refer the daughter to drug rehabilitation D) explore the issues as they relate to his work performance

Answer is D. In this case, the social worker provides job-related counseling. If she explores job related issues first it is possible the other options may be addressed. When working for an EAP, your ultimate responsibility belongs to the employer.

A social worker is assigned a client in the hospital to assist with providing information about a living will. After assessment and according to the record the social worker agrees the client is competent to make this decision. What should the social worker do to assist the client: A. speak with the family B. refer to an attorney to assist the client C. get a second opinion from the medical staff working with her D. assist the client with her wishes

Answer is D. In this question, taking into account the choices, the client is competent, we assist the client. This will most likely involve helping the client to talk with the family and helping them process. It is the decision of the client. The two best choices are A and D. I also would say, you need permission from the client to go to the family in the first place.

Parents in neighborhood cannot afford daycare services. A local coalition has been formed and it concurs with a needs assessment that shows the need for affordable daycare. However, the community group wants to put their energy into providing a meals-on-wheels service instead. What can the social worker, who is part of the team, do to help the support the need for more affordable daycare? A) suggest that the committee revisit their focus B) ask the committee to justify their selection of the service C) threaten to resign from the committee D) suggest the committee address and prioritize the goals identified in the needs assessment

Answer is D. It is best to get the group to prioritize goals from the needs assessment.

A large heath care agency has just put into affect plans to expand their operations taking affect five years from now. This is an example of: A) structural functional theory B) strategic functional theory C) management by objectives D) policy planning implementation

Answer is D. It is not A or B or C, so it has to be D.

A social worker is assigned a case that involves an immigrant client with a disabled child. The social worker has not met the client yet but has found a resource to help the family with respite for the disabled daughter she wants to share. What is the best way to provide this referral? A) call the family and tell them about the program. B) give name and number of respite program to family. C) call the family to ask them if they would like the information D) make an appointment to meet the family and present the information at that time.

Answer is D. Make an appointment and establish rapport with the family as they may be more accepting of the referral.

A couple is seen in treatment for problems with impotency. The wife insists that husband get treatment or she will leave him. He agrees to do this. After he gets treatment the problem with impotency is resolved although he is shocked to find that his wife leaves him anyway. This is most likely due to: A) the husband had more problems than impotency B) the sexual problem had more to do with the wife than the husband C) when treating sexual problems transactional interactions should be explored D) the impotency was only the presenting problem

Answer is D. Many time couples present with one problem but it is merely the presenting problem or what has brought them in for treatment.

A woman and her husband are Muslims and see a social worker for counseling. The wife wears a face covering and the social worker cannot always see her responses to what is being communicated in the session. What should the social worker do? A) tell her to uncover her face B) let the spouse answer for her C) refer her to another social worker D) tell her you cannot see her responses

Answer is D. She cannot remove her veil. It is possible for the husband to answer for her but let her make that decision. From these choices it is best to tell her you cannot see her responses.

To avoid personal biases due to preconceptions about a client's race, gender, sex, or culture in conducting an initial interview, a social worker should first focus on: A) personal experiences and common sense B) definition of the problem C) theories of behaviors and data on presenting problems D) clients experience and definition of the problem

Answer is D. Start where the client is.

In the inpatient unit of a large community hospital, a client seeing a social worker reports feeling depressed, sad lost and feeling alone. He tells the social worker he is HIV positive and to help combat his loneliness he has had several new sexual partners. He gives the social worker the name of his partners but states he has no intention on telling them he is HIV positive. The social worker encourages him to tell them but he refuses to consider it. The hospital has an infectious disease unit. What should the social worker do next? A) Report the information to the local Health Department, for the clients own health and safety B) Request patient sign a release of information so the social worker can notify the partners C) Report client to law enforcement so the partners can be notified D) Report the case and her concerns directly to the infectious disease nurse

Answer is D. Start with what it is not. It is not C, we should not notify the police, what are they going to do? Also, they cannot notify the partners. In this sample question, it is not clear if the state requires mandatory reporting of this or not. In some states, it is a misdemeanor not to report. Answer B, signed release or not, this is not the role of the social worker to notify the partners. This is better handled by the health department or those directly involved in addressing the spread of infectious diseases. Answer is D, as the infectious disease nurse is a mandatory reporter. It is between A and D, but it is better to go through the proper channels than to go outside to the health department at this point.

Answer is D. We sometimes see this in children that have been left to go hungry, etc. Another choice might be B, but before you go there I would first consider that it could be normal behavior for her based on where she had been living before.

Answer is D. Tardive dyskinesia

A young women reports to her social worker that she has just broke up with her boyfriend. The woman tells the worker since the break-up she is experiencing a loss of appetite, has problems sleeping, is irritable and having stomach problems. She reports crying since the break-up and has an inability to focus and pay attention. According to the DSM-IV-TR would most likely be diagnosed with: A) major depressive disorder B) bipolar one disorder C) early onset schizophrenia D) adjustment disorder with depressed mood

Answer is D. The answer is D. To find the answer, start with what it is not and work backwards. It is not C and it is not B. This leaves A and D. This depression appears to be related to the loss of the boyfriend; therefore, adjustment disorder seems the most appropriate choice.

A social worker is in the initial intake interview with a male client who reports having problems with his anger. He admits that he does drink alcohol on occasion and is not sure if it is related to his substance use. The social worker asked for the client to tell her a time he has gotten angry. He says the last time it happened he was with his wife and was not drinking alcohol but got so angry with her he almost hit her. As part of her recommendations, what is the first thing the social worker should suggest? A) recommend the client immediately get help for his substance abuse B) explain to the client the negative affect alcohol is having on him and his family situation C) tell him that you must report the abuse D) recommend that he consider counseling for his angry outbursts

Answer is D. The best recommendation at this point is to address the anger outbursts since they seem to occur when he is not drinking.

A worker has been summoned to appear in court and ordered to hand over his documents of a former client. The worker refuses to release the record as requested. A) it is the workers right to refuse without penalty, on the grounds of conflict of interest B) the worker will automatically have his license suspended and will be fined for failure to submit court ordered documents C) the worker is protected under client confidentiality D) the worker could have legal charges brought against him

Answer is D. The worker must comply with a court order. If the worker decides to assert privilege and not release the record, he or she has the right to go to jail. Legal charges can be filed against the social worker and jail time is possible.

You are trying to see if your client is able to receive assistance and beneficial treatment from the Native American care unit located in your area. The tribes' chief oversees the program that offers services to Native Americans without charge. The social worker should first: A) ask to obtain a birth certificate of your client to see if he is truly Native American B) the social worker needs to prove he is at least 1/4 Native American before she can refer C) bring in some of his family that is Native American to verify the client's heritage D) refer the client for treatment and let the reservation decider

Answer is D. There are two ways to qualify...belong to a registered tribe or have the verification of blood relationships. I am not sure exactly how much that is as the law itself is not clear on how much blood, I believe it is at least 25%. From the choices above I would say D, refer and let the reservation decide. The tribe will make the decision.

When working with a client suffering from intermittent-explosive disorder, of the four choices below, which one changes the diagnosis? A) depression B) alcohol abuse C) violent behavior toward others D) self-control

Answer is D. These individuals lack self-control and become violently angry and explosive.

A couple seeks services to enhance their relationship. After therapy the couple feels divorce is best. The social worker states the purpose of therapy is to improve the relationship, not end it. The social worker's statement: A) best addresses the workers attempts the clients needs B) influences the workers personal values on the process C) need to address the client's ambivalence about their relationship D) is used to refocus the objective of their therapy

Answer is D. This is a strange thing to say but sometimes making changes is hard work and people want to run away from it and simple answer is divorce. The social worker may be refraining and clarifying for them.

A woman with two teen age children is unable to cope with her life situation and is placed in a mental hospital. She has a history of physical and emotional problems. She also has dysthymia. The best theory to help her understand her history of problems is: A) ecological B) rational emotive C) psychoanalytical D) problem solving

Answer is D. This is best, followed by B. As if she has dysthymia this condition is high functioning and capable of rational thought and problem solving.

The father of a six-year-old is teaching the child to throw a baseball. Each time the child throws the ball correctly, the father praises him, smiles, and claps his hands. When the child fails to throw the ball as instructed, the father is silent. The father's praise is and example of? A) strategic reinforcement B) partial reinforcement C) negative reinforcement D) selective positive reinforcement

Answer is D. This is positive reinforcement that is given selectively.

In a mental health facility the type, and amount of care that's provided to the client is determined by A) the social worker handling the case B) psychologist assigned to each client C) a committee made up of clients D) specialized professionals trained to manage resources

Answer is D. This team will determine what is covered.

A group of social workers are working on formulating a legislative bill to submit for approval. The first step they should take is. A) get the media to help get the public behind there Bill and efforts. B) submit the Bill to a local official C) organize to ensure the Bill is written and formulated properly D) get local officials involved in creating the Bill before you submit it

Answer is D. This type of grass roots work will be the first step in getting the bill ready to submit. It will also help to make sure it is reflective of the community needs.

An eight-year old girl has recently been adopted after leaving a traumatic environment where she lived with her drug addicted mother. Her adoptive parents are concerned as she is hoarding food in her room. What should the social worker assess for first: A) possible sexual abuse as a toddler as this could be a sign of sexual abuse B) a sign of a future eating disorder C) normal behavior for all children D) possible normal behavior for a child recently adopted having come from a chaotic environment

Answer is D. We sometimes see this in children that have been left to go hungry, etc. Another choice might be B, but before you go there I would first consider that it could be normal behavior for her based on where she had been living before.

A 30-year-old male client is being seen by a social worker under a court order after being mandated to treatment by a judge to help the client control his anger outbursts. After discussing a difficult topic in his last session, the client calls the office and states on the answering machine that he feels he does not need treatment anymore and is not returning next week. What should the social worker do first? A) call him back and insist he return as it is a court ordered treatment B) call him the next day and see if he feels differently about his decision C) acknowledge the rights of the client for declining the treatment D) inform the court when/if he discontinues therapy

Answer is D. When a client is mandated to complete therapy and they discontinue the court will need to be notified.

In the first session a social worker is working with an elderly Asian woman. To start the session the social worker should: A) move closer and sit next to the client B) make immediate eye contact with the client C) say nothing and let the client start the conversation with the social worker D) introduce herself and ask the client how she would like to be addressed

Answer is D. With Asian clients the social worker should always treat them with respect and honor. Once the client makes eye contact and moves closer the social worker has permission to respond. At all times treat the client with respect.

A social worker is developing a group with depressed older adults who are having a hard time dealing with retirement. What should the social worker do first? A) decide what treatment modalities should be used B) go over prepared slides in group very slowly C) continue sameness and repetition of all information to make sure they understand D) work with the group to mutually decide the goals of the group

Answer is D. Work with the group to develop group goals. Once this has been done the others choices can be explored.

A social worker is working with a client when she hears the physician say that the medication the client is taking is paradoxical. Which of the following medications is considered paradoxical? A) Prozac B) lithium C) Ritalin D) Melaril

Answer is Ritalin. It is a stimulant that is used to help children calm and think more clearly.

Which indicates the extreme avoidance of feelings, where the person becomes unaware of why he or she is angry? A) repression B) regression C) denial D) sublimation

Answer is a repression.

Which of the following of the communication techniques most displays underlying meaning? A) silence B) validation C) open ended question D) confrontation

Answer is a, silence.

A client presents with a self-centered attitude and is perceived as aggressive in his contacts with others. He has been in jail for numerous offenses with his last being robbery and kidnapping having taken a hostage to facilitate his escape. Which personality disorder most often exhibits these traits? A) antisocial B) narcissistic C) dependent D) schizoid

Answer is antisocial personality disorder. The second choice might be narcissistic but antisocial more clearly matches the criteria given in the description.

According to psychodynamic theory, when a client is of a different race from the therapist, this would most likely preclude: A. transference B. counter-transference C. therapeutic initiatives D. discrimination

Answer is. The two best choices are A or B, and either could occur but which one is MOST likely. As a trained therapist, the professional would be most aware that cultural perceptions and awareness could be a problem. So therefore, it would be most likely the client would experience transference and be unaware of why this was happening.

A client is in the third session with a social worker and reveals that he is HIV positive. The social worker is in private practice and is seeing the client to work on issues related to his depression and his partner leaving him. In one session the client names several partners with whom he has had sexual relations. The client says he does not want the new partners to know of his condition because he believes it will change the way they look at him. The social worker encourages him to notify his partners. The client refuses. In a state where no reporting is required legally to the health department or to the at risk victims, what should the social worker do first? A) Report the incident/information to the health department and ask whether it needs to be reported. B) Try to get written permission from client to notify the named persons. C) Try to contact the named persons without the client's permission. D) Ask the client to notify the health department.

Answer: A. A physician is a mandated reporter and so is any lab, etc. that has the positive result. For these individuals they can loose their license to practice if they do not report. For other professionals, it does depend on the state, in some states it is a misdemeanor to not report to the health department. Almost every state has reporting laws related to this and for a social worker it is critical to find out what they are. Generally, if you are unaware of the policy call the health department and find out. All states have a number to call and a system for reporting. Also, be sure you know what diseases fall into this area as it can include all infections diseases such as syphilis, infectious stage hepatitis, etc.

A client is in the third session with a social worker and reveals that he is HIV positive. The social worker is in private practice and is seeing the client to work on issues related to his depression and his partner leaving him. In one session the client names several partners with whom he has had sexual relations. The client says he does not want the new partners to know of his condition because he believes it will change the way they look at him. The social worker encourages him to notify his partners. The client refuses. In a state where no reporting is required legally to the health department or to the at-risk victims, what should the social worker do first? A) Call the health department and ask what can be done in a situation of this type to help protect the partners B) Try to get written permission from client to notify the named persons. C) Try to contact the named persons without the client's permission. D) Ask the client to notify the health department.

Answer: A. A physician is a mandated reporter and so is any lab, etc. that has the positive result. For these individuals they can loose their license to practice if they do not report. For other professionals, it does depend on the state, in some states it is a misdemeanor to not report to the health department. Almost every state has reporting laws related to this and for a social worker it is critical to find out what they are. Generally, if you are unaware of the policy call the health department and find out. All states have a number to call and a system for reporting. Also, be sure you know what diseases fall into this area as it can include all infections diseases such as syphilis, infectious stage hepatitis, etc..

A social worker receives a handmade afghan as a gift from an elderly client. The client says she wants to give the social worker the gift to say thank you for all she has done for her and her family. What should the social worker do? A) accept gift for herself on behalf of the organization B) thank the client but explain you cannot take the gift for your own personal use C) ask client to give the gift to another agency D) before you respond to the client, call your supervisor and ask what the policy for accepting gifts is

Answers is B. Thank the client and explain that in social work we generally do not accept gifts from clients. One exception might be if it is important to do so for cultural reasons and another is if there is no value attached to the gift. If you do accept a gift, make sure that you do not use it for personal use at home, etc.

A social worker is working with a parent who has a child that is being considered for the diagnosis of ADHD. The social worker correctly tells the parent that the onset of the disorder is before: A) age 5 B) age 7 C) age 3 D) age 12

Correct answer is B and some symptoms must be evident before age 7. This is being debated but it still represents the current criteria. Some professionals argue that it is later than that, especially for the inattentive type. Also, it is problematic in adults to really identify that this was a problem back in childhood, especially with the adult ADHD (inattentive) type.

A man is being evaluated at a hospital for suicidal thoughts. He denies any thoughts of suicide saying that he does not want to harm himself and that he has never felt depressed. His girlfriend reports that he has been sad for the last two weeks and she is concerned about him. The social worker is convinced that the man is not a threat to himself but remains concerned about the reported guns that are in the home. The social worker should: A) ask the girlfriend to lock up the guns B) ask the police to search the home C) allow him to make a decision on his own D) seek immediate hospitalization

If there is concern for safety and discharge is eminent, a safety plan should be clearly implemented. The social worker, even though she feels he is not a danger to self or others, is wise to set up a safety plan upon discharge especially since the family is concerned. Look at the choices, which ones come closest to a safety plan. Answer A, is one step toward a plan. It would be better to ask him to work with the girlfriend and put away the guns, but this is not a choice. Best response is A, followed C, as he is mentally competent to make this decision.


Ensembles d'études connexes

ATI PROCTORED EXAM - MATERNAL NEWBORN

View Set

Correlation vs. Causation: Differences & Definition

View Set

Questions with "tu aimes" (Safia)

View Set

七上历史chapter4 section 4 the new kingdom and kush

View Set

Module 15 Programming and Scripting

View Set